Gastrointestinal Imaging Flashcards

1
Q

Liver Anatomy

A

The Couinaud classification divides the liver into eight segments. Because each segment is self-contained, an individual segment can be completely resected without disturbing the other segments.

Numbering of hepatic segments is clockwise when looking at a frontal/coronal view.

Each hepatic segment features its own: (Central portal triad including branches of the portal vein, hepatic artery, and bile duct. Peripheral venous drainage to the hepatic veins and ultimately the IVC.)

The caudate lobe drains directly to the IVC, not into the hepatic veins. The caudate lobe is spared in early cirrhosis since the direct drainage to the IVC spares the caudate from increased venous pressures due to portal hypertension. This leads to compensatory hypertrophy of the caudate lobe, which is a typical morphologic change of early cirrhosis.

How well did you know this?
1
Not at all
2
3
4
5
Perfectly
2
Q

Liver CT

A

A “routine” contrast-enhanced abdominal CT is acquired in the portal venous phase of enhancement, typically obtained 70 seconds following intravenous contrast administration.

A portal venous phase CT reveals characteristic attenuation alterations and/or morphologic changes of diffuse liver disease, such as hepatic steatosis and cirrhosis. Most metastatic tumors are not hypervascular (although a few notable exceptions will be subsequently discussed) and can also generally be detected on the portal venous phase. Of note, rarely some breast cancers may be isoattenuating on the portal venous phase and more conspicuous on unenhanced CT.

Most benign and malignant primary liver masses are hypervascular and thus most conspicuous in the arterial phase of enhancement. The arterial phase begins approximately 20-25 seconds after intravenous contrast injection. Many authors advocate that optimal conspicuity of a hypervascular liver lesion is obtained in the late arterial phase, which is between 9 and 16 seconds after abdominal aortic enhancement, or approximately 35 seconds after intravenous injection.

How well did you know this?
1
Not at all
2
3
4
5
Perfectly
3
Q

Liver MRI

A

Compared to CT, MRI of the liver displays the same patterns of contrast enhancement but has superior lesion-to-liver contrast. MRI also does not impart ionizing radiation.

MRI is able to obtain dynamic post-contrast images in multiple phases without any penalty in radiation exposure. In- and out-of-phase gradient imaging allows detection of intractyoplasmic lipid, which is seen in hepatic steatosis. Additionally, advanced techniques such as diffusion-weighted imaging may show potential for clinical use.

How well did you know this?
1
Not at all
2
3
4
5
Perfectly
4
Q

Fatty liver (hepatic steatosis)

A

Nonalcoholic fatty liver diease can be divided into steatosis and steatosis with associated inflammatory activity (steatohepatitis). Overall, greater than 15% of the population is afflicted with nonalcoholic fatty liver disease (NAFLD), which is a component of the metabolic syndrome of obesity, insulin resistance, and dyslipidemia. Ultimately, steatohepatitis may progress to cirrhosis.

CT can determine if steatosis is present and can provide a rough gauge as to its severity. In- and out-of-phase MRI imaging can more accurately quantify the degree of steatosis, although liver biopsy is the gold standard and best evaulates for the presence of inflammatory and early fibrotic change. By the time imaging can detect morphologic changes of cirrhosis, the changes may be irreversible.

On unenhanced CT, the liver should be slightly hyperattenuating relative to the spleen. The traditional teaching is that steatosis is present if the liver attenuates at least 10 Houndsfield units (HU) less than the spleen, although new work suggests that even a single HU of relative hypoattenuation compared to the spleen may represent hepatic steatosis.

On contrast-enhanced CT, evaluation of hepatic steatosis is much less reliable compared to unenhanced CT due to different contrast uptake rates of the liver and the spleen. However, the liver is considered diffusely hypoattenuating if it attenuates at least 25 HU less than the spleen in the portal venous phase.

In- and out-of-phase GRE MRI is a sensitive imaging technique to evaluate for the presence of (and to quantify the degree of) hepatic steatosis.

Variations in portal venous supply may cause geographic regions that are more or less affected by fatty change. Focal fat does not have any mass effect, vessels characteristically run through it, and it tends to occur in the following typical locations.

How well did you know this?
1
Not at all
2
3
4
5
Perfectly
5
Q

Amyloid

A

Abnormal extracellular deposition of amyloid protein in the liver can cause focal or diffuse areas of decreased attenuation on CT imaging.

How well did you know this?
1
Not at all
2
3
4
5
Perfectly
6
Q

Wilson disease

A

Wilson disease causes high levels of copper to accumulate in the basal ganlgia, cornea, and liver due to an autosomal recessive genetic defect. The liver may be hyperattenuating on CT with multiple nodules, eventually leading to hepatomegaly and cirrhosis.

How well did you know this?
1
Not at all
2
3
4
5
Perfectly
7
Q

Hepatic Iron Overload

A

There are two pathways to excess hepatic iron accumulation. Accumulation within hepatocytes is seen in hemochromatosis. Uptake within the reticuloendothelial system (RES) causes hepatic Kupffer cell iron overload, as seen in hemosiderosis. Regardless of the etiology, the iron-overloaded liver is hypointense on all MRI sequences, relative to the paraspinal muscles as an internal control. Hemochromatosis is the most common cause of iron overload, due to a genetic defect causing increased iron absorption. Excess iron is unable to be stored in the RES, so the spleen and bone marrow are not affected. Treatment of hemochromatosis is phlebotomy.

Hemosiderosis is excess iron stored within the reticuloendothelial system, which may be due to frequent blood transfusions or defective erythrocytosis. Treatment of hemosiderosis is with iron chelators, not phlebotomy.

Hemosiderosis is excess iron stored within the reticuloendothelial system, which may be due to frequent blood transfusions or defective erythrocytosis. Treatment of hemosiderosis is with iron cheltors, not phlebotomy.

Hemosiderosis is a precursor to secondary hemochromatosis. Secondary hemochromatosis is hepatic damage from iron overload after the RES system becomes saturated from prolonged hemosiderosis.

In clinical practice, the distinction between hemosiderosis and secondary hemochromatosis often overlaps. Many authors recommend against the term “secondary hemochromatosis” in favor of describing the primary disease (e.g., thalassemia) with secondary iron overload.

Hypoattenuating liver: The liver is considered hypoattenuating if it attenuates less than the spleen on an unenhanced CT. (Fatty liver (hepatic steatosis) is by far the most common cause of a diffusely hypoattenuating liver. Hepatic amyloid is rare and may cause either focal or diffuse hepatic hypoattenuation.)

Hyperattenuating liver: The normal unenhanced attenuation of the liver is 30 to 60 HU. An absolute attenuation greater than 75 HU is considered hyperattenuating. (Iron overload is by far the most common cause of a hyperattenuating liver. Medications (e.g. amiodarone, gold, and methotrexate). Copper overload (Wison disease). Glycogen excess)

How well did you know this?
1
Not at all
2
3
4
5
Perfectly
8
Q

Viral Hepatitis

A

Patients with viral hepatits often have a normal CT scan. Viral hepatitis may cause nonspecific CT findings, such as gallbladder wall thickening or periportal edema (fluid on both sides of the portal veins).

How well did you know this?
1
Not at all
2
3
4
5
Perfectly
9
Q

Candidiasis

A

Systemic fungal infection may seed the liver (and commonly the spleen as well) due to portal venous drainage of infected bowel. CT shows multiple tiny hypoattenuating microabscesses in the liver and the spleen, which may be rim-enhancing.

Candidiasis is almost always seen in immunocompromised patients.

The differential diagnosis for multiple tiny hypoattenuating hepatic lesions includes metastatic disease, lymphoma, biliary hamartomas, and Caroli disease.

How well did you know this?
1
Not at all
2
3
4
5
Perfectly
10
Q

Abscess

A

Hepatic abscess is most commonly caused by a bowel process and resultant infectious nidus carried through the portal system to the liver. Common causes include diverticulitis, appendicitis, Crohn disease, and bowel surgery. E. coli is the most common organism. A primary hepatobiliary infection, such as ascending cholangitis, may be a less common cause.

Imaging features of hepatic abscess may mimic metastasis, appearing as a ring-enhancing mass on CT. On MRI, there is typically central hyperintensity on T2-weighted images with an irregular wall that enhances late. Perilesional enhancement may be present.

How well did you know this?
1
Not at all
2
3
4
5
Perfectly
11
Q

Echinococcal disease

A

Hepatic echinococcus is caused by ingestion of the eggs of Echinococcus granulosus, which is endemic in the Mediterranean basin and associated with sheep-raising. Echinococcal eggs can develop into hydatid cysts.

On CT, a hydatid cyst is a well-defined hypoattenuating mass featuring a characteristic floating membrane or an associated daughter cyst. Peripheral calcification may be present.

How well did you know this?
1
Not at all
2
3
4
5
Perfectly
12
Q

Cirrhosis

A

Cirrhosis is caused by repeated cycles of injury and repair, which can be due to metabolic (alcohol, steatohepatitis, hemochromatosis, or Wilson disease), infectious (chronic hepatitis B or C), or inflammatory (primary biliary cirrhosis or primary sclerosing cholangitis) etiologies. The hallmarks of cirrhosis are fibrosis and attempted, disorganized regeneration.

The micronodular form of cirrhosis is most often due to metabolic causes.

The macronodular form of cirrhosis is most often post-viral (hepatitis B or C).

One of the earliest signs of cirrhosis is expansion of the preportal space. Atrophy of the medial segment of the left hepatic lobe in early cirrhosis causes increased fat anterior to the right main portal vein. Enlargement of the caudate lobe is a specific sign of cirrhosis. Specifically, a caudate to right lobe size ratio >0.65 highly suggests cirrhosis. As previously discussed, the caudate drains directly to the IVC, not via the hepatic veins, which results in compensatory caudate hypertrophy. The empty gallbladder fossa sign results when hepatic parenchyma surrounding the gallbladder is replaced with periportal fat.

Portal hypertension causes splenomegaly, portosystemic collaterals, and varices. Gallbladder wall thickening is due to hypoalbuminemia and resultant edema. Gamna-Gandy bodies are splenic microhemorrhages, which appear hypointense on GRE.

How well did you know this?
1
Not at all
2
3
4
5
Perfectly
13
Q

Pathway to hepatocellular carcinoma

A

In the setting of cirrhosis, hepatocellular carcinoma (HCC) is thought to develop in a sequence from regenerative nodule to dysplastic nodule to HCC. Regenerative and dysplastic nodules cannot be reliably differentiated on imaging; and high-grade dysplastic nodules cannot be reliably differentiated from low-grade HCC.

Regenerative nodule: A regenerative nodule is completely supplied by the portal vein and is not premalignant. A regenerative nodule should not enhance in the arterial phase. (Most regenerative nodules show low signal intensity on T2-weighted images, with variable signal intensity on T1-weighted images. Rarely, a regenerative nodule may be hyperintense on T1-weighted images due to glycogen deposition. On contrast-enhanced MRI, most regenerative nodules enhance to the same (or slightly less) degree as the adjacent hepatic parenchyma.)

Dysplastic nodule: Unlike a regenerative nodule, a dysplastic nodule is premalignant. However, most dysplastic nodules do not demonstrate arterial phase enhancement (unless high grade), since blood supply is still from the portal vein. (Dysplastic nodules are variable in signal intensity on T1-weighted images. Most dysplastic nodules are hypointense on T2-weighted images, although high-grade dysplastic nodules may be T2 hyperintense. Contrast-enhanced MRI shows low-grade dysplastic nodules to be iso-enhancing relative to liver and thus indistinguishable from regenerative nodules. High grade dysplastic nodules may demonstrate arterial enhancement and be indistinguishable from well-differentiated hepatocellular carcinoma.)

A siderotic nodule is an iron-rich regenerative or dysplastic nodule. A siderotic nodule is hypointense on T1 and T2-weighted images and hyperattenuating on CT. A siderotic nodule is rarely, if ever, malignant.

How well did you know this?
1
Not at all
2
3
4
5
Perfectly
14
Q

Hepatocellular Carcinoma

A

Hepatocellular carcinoma (HCC) is the most common primary liver tumor. Cirrhosis is the major risk factor for development of HCC. A hypervascular liver mass in a patient with cirrhosis or chronic hepatitis is an HCC until proven otherwise.

Alpha-feto protein (AFP) is elevated in approximately 75% of cases of HCC.

Arterial phase enhancement is the characteristic imaging feature of HCC. However, between 10 and 20% of HCCs are hypovascular and thus slightly hypoenhancing relative to surrounding liver on arterial phase imaging.

The classic CT or MRI appearance of HCC is an encapsulated mass that enhances on arterial phase and washes out on portal venous phase. HCC may be difficult to detect on non-contrast or portal venous phase CT. On unenhanced MRI, HCC is characteristically slightly hyperintense on T2-weighted images relative to surrounding liver. (The nodule in a nodule appearance describes an enhancing nodule within a dysplastic nodule and represents an early HCC).

HCC is locally invasive and tends to invade into the portal and portal veins, IVC, and bile ducts. In contrast, metastases to the liver are much less likely to be locally invasive.

Treatment options for HCC include partial hepatectomy, orthotopic liver transplantation, percutaneous ablation, and transcatheter embolization.

How well did you know this?
1
Not at all
2
3
4
5
Perfectly
15
Q

Fibrolamellar HCC

A

Fibrolamellar carcinoma is a subtype of HCC that occurs in young patients without cirrhosis.

The tumor tends to be large when diagnosed, but has a better prognosis than typical HCC. Unlike in HCC, AFP is not elevated.

On MRI, fibrolamellar HCC is a large, heterogenous mass. A fibrotic central scar is classic, which is hypointense on T1 and T2-weighted images (in contrast, focal nodular hyperplasia feature a T2 hyperintense scar that enhances late). Capsular retraction may be seen in 10%.

Unlike HCC, the fibrolamellar subtype does not have a capsule, although there may be a pseudocapsule of peripherally compressed normal hepatic tissue.

How well did you know this?
1
Not at all
2
3
4
5
Perfectly
16
Q

Hepatic Metastases

A

Although metastases are supplied by branches of the hepatic artery induced by tumoral angiogenesis, most metastases are hypovascular and best appreciated on portal venous phase (in contrast to HCC, which is hypervascular and best visualized on late arterial phase).

Hypervascular metastases (best seen on arterial phase) classically include: (Neuroendocrine tumors, including pancreatic neuroendocrine tumors and carcinoid. Renal cell carcinoma. Thyroid carcinoma. Melanoma. Sarcoma.)

Colorectal and pancreatic adenocarcinoma metastases are typically hypovascular and can be usually diagnosed on portal venous imaging.

Calcifications can be seen in mucinous colorectal tumors or ovarian serous tumors. Calcification within a metastatic lesion may imply a better prognosis.

On MRI, metastatic lesions tend to be hypointense on T1-weighted images and hyperintense on T2-weighted images. Blood products and melanin (as in melanoma) are T1 hyperintense.

Pseudocirrhosis describes the macronodular liver contour resulting from multiple scirrhous hepatic metastases, which may mimic cirrhosis. Treated breast cancer is the most common cause of this appearance. Capsular retraction, although not always seen, is characteristic of pseudocirrhosis, and when present suggests pseudocirrhosis over cirrhosis.

How well did you know this?
1
Not at all
2
3
4
5
Perfectly
17
Q

Hepatic lymphoma

A

Primary hepatic lymphoma is very rare. Lymphomatous involvement of the liver tends to be secondary to systemic disease, with associated splenomegaly and lymphadenopathy.

How well did you know this?
1
Not at all
2
3
4
5
Perfectly
18
Q

Epithelioid hemangiothelioma

A

Epithelioid hemangioendothelioma is a rare vascular malignancy that characteristically causes multiple spherical subcapsular masses that can become confluent. The individual masses may have a halo or target appearance. Epithelioid hemagioendothelioma is one cause of capsular retraction.

How well did you know this?
1
Not at all
2
3
4
5
Perfectly
19
Q

Hepatic capsular retraction

A

Capsular retraction is a focal convexity of the normally convex external liver contour.

Metastatic tumor (more commonly post-treatent)

Fibrolamellar hepatocellular carcinoma (10% of cases of fibrolamellar HCC).

Hepatocellular carcinoma (capsular retraction has been reported but is uncommon).

Epithelioid hemangioendothelioma.

Intrahepatic cholangiocarcinoma.

Confluent hepatic fibrosis (wedge-shaped fibrosis seen in cirrhosis, most commonly the medial segment of the left hepatic lobe or the anterior segment of the right hepatic lobe).

How well did you know this?
1
Not at all
2
3
4
5
Perfectly
20
Q

Focal Nodular Hyperplasia

A

Focal Nodular Hyperplasia (FNH) is disorganized liver tissue with no malignant potential. It is primarily seen in asymptomatic women and is not associated with oral contraceptives.

FNH has a characteristic central “scar” which does not contain fibrotic tissue and is therefore not a true scar. Instead, the central area consists of T2-hyperintense ductules and venules, and demonstrates delayed enhancement. FNH does not have a capsule.

FNH can be difficult to see without contrast on CT and T1- and T2-weighted MRI sequences. FNH avidly enhances during the arterial phase, then washes out very quickly. The portal venous phase will often show just the unenhanced scar, which enhances late.

Kupffer cells and bile duct epithelium are both present. Kupffer cells may be confirmed by a sulfur colloid study (1/3 of the time) and bile duct cells can be seen on a HIDA scan.

How well did you know this?
1
Not at all
2
3
4
5
Perfectly
21
Q

Hemangioma

A

A hepatic hemangioma is a benign mass composed of disorganized endotheliallined pockets of blood vessels, supplied by a branch of the hepatic artery at the periphery.

Hemangioma is more common in females and uncommon in cirrhosis. When a known hemangioma is sequentially followed in a patient with early cirrhosis, the hemangioma involutes as the liver becomes more cirrhotic.

Hemangiomas may range in size from <1 cm to >10 cm. Giant hemangiomas tend to have a nonenhancing central area representing cystic degeneration.

A virtually pathognomonic imaging feature is periphera, discontinuous, progressive, nodular enhancement. The attenuation (or signal intensity on MR) of the enhancement is identical to the aorta and features gradual centripetal fill-in on later phases.

The unenhanced CT appearance of a hemangioma is a nonspecific hypoattenuating liver mass.

How well did you know this?
1
Not at all
2
3
4
5
Perfectly
22
Q

Hepatic Adenoma

A

Hepatic adenoma is a benign hepatic neoplasm containing hepatocytes, scattered Kupffer cells, and no bile ducts.

Adenomas are much more common in females, especially with prolonged oral contraceptive use. When seen in males, adenoma may be associated with anaboic steroids.

Adenomas have a relatively high risk of hemorrhage, which is often the presenting symptom. For this reason, incidentally discovered adenomas are usually resected.

Multiple hepatic adenomas are seen in von Gierke disease (type I glycogen storage disease).

A pseudocapsule may be present, which tends to enhance late.

Adenomas lack portal venous drainage and thus are hypervascular on arterial phase. The presence of microscopic fat, when present, is best seen on in- and out-of-phase MRI. Intralesional hemorrhage may cause T1 hyperintensity. Adenomas may be difficult to differentiate from other hypervascular liver lesions in the absence of fat or hemmorrhage.

How well did you know this?
1
Not at all
2
3
4
5
Perfectly
23
Q

Budd-Chiari

A

Budd-Chiari is hepatic venous outflow obstruction, which can be thrombotic or non-thrombotic. Budd-Chiari may be due to hypercoagulative states including hematological disorders, pregnancy, oral contraceptives, malignancy, infection, and trauma. It is very rare to have primary Budd-Chiari due to congenital hepatic veins anomaly, as pictured below.

Acute Budd-Chiari presents with a clinical triad of hepatomegaly, ascites, and abdominal pain.

Direct vascular findings include lack of flow within hepatic veins, thrombus in the hepatic veins/IVC, and the formation of collateral vessels.

Acute intraparenchymal findings include an edematous peripheral liver with sparing of the caudate lobe. The caudate is spared as it drains directly into the IVC.

Progressive liver failure may result in chronic disease, producing caudate lobe hypertrophy and atrophy of peripheral liver with prominent regenerative nodules.

How well did you know this?
1
Not at all
2
3
4
5
Perfectly
24
Q

Veno-occlusive diease

A

Veno-occlusive disease (VOD) is destruction of post-sinusoidal venules, with patent hepatic veins. VOD is seen in bone marrow transplant patients, possibly due to chemotherapy.

Imaging findings are nonspecific. Periportal edema, narrowing of the hepatic veins, hepatomegaly, and heterogenous hepatic enhancement have been reported. In contrast to Budd-Chiari, the caudate lobe is not spared.

How well did you know this?
1
Not at all
2
3
4
5
Perfectly
25
Q

Cardiac Hepatopathy

A

Cardiac hepatopathy is passive hepatic congestion from heart failure, constrictive pericarditis, or right-sided valvular disease, which ultimately may lead to cirrhosis.

Imaging clues are enlarged hepatic veins and IVC, with reflux of intravenous contrast from the right atrium into the IVC and hepatic veins. The liver is typically enlarged and demonstrates mottled enhancement. Ascites is usually present.

How well did you know this?
1
Not at all
2
3
4
5
Perfectly
26
Q

Biliary hamartomas (von Meyenburg complexes)

A

Biliary hamartomas are incidental small cystic lesions that do not communicate with the biliary tree, caused by embryologic failure of normal bile duct formation.

Biliary hamartomas tend to be smaller and more irregularly shaped than simple cysts.

How well did you know this?
1
Not at all
2
3
4
5
Perfectly
27
Q

Autosomal dominant polycystic liver disease (ADPLD)

A

40% of patients with autosomal dominant polycystic kidney disease (ADPKD) have a similar disease process in the liver, called ADPLD. Even in severe disease, hepatic failure is rare.

On imaging, there are innumerable nonenhancing simple cysts throughout the liver.

How well did you know this?
1
Not at all
2
3
4
5
Perfectly
28
Q

Liver Trauma

A

The liver is the second most commonly injured solid organ due to blunt trauma, second to the spleen. The CT description and grading of liver injury is similar to splenic injury.

The American Association for the Surgery of Trauma (AAST) classification describes hepatic injury based on findings at laparotomy.

The MDCT injury grading scale is based on CT findings and is more commonly used by radiologists. It is similar to the MDCT grading scale for splenic trauma.

How well did you know this?
1
Not at all
2
3
4
5
Perfectly
29
Q

MDCT hepatic trauma

A

Grade 1: Superficial laceration or subcapsular hematoma <1 cm in size.

Grade 2: Laceration or subcapsular/intraparenchymal hematoma >1 and <3 cm in size.

Grade 3: Laceration or subcapsular/intraparenchymal hematoma >3 cm in diameter.

Grade 4: Massive hematoma >10 cm, or destruction/devascularization of one hepatic lobe.

Grade 5: Destruction or devascularization of both hepatic lobes.

How well did you know this?
1
Not at all
2
3
4
5
Perfectly
30
Q

MRCP

A

Magnetic resonance cholangiopancreatography (MRCP) is an abdominal MRI acquired with heavily T2-weighted sequences that increase the contrast between T2 hyperintense stationary fluid in the biliary tract and surrounding structures.

Fast spin echo sequences are most commonly used for MRCP acquisition. Various techniques can be employed to optimize imaging including breath-hold sequences and respiratory-triggered sequences.

Heavily T2-weighted sequences primarily image the biliary tree.

Sequences with intermediate T2 (TE 80-100ms) are best suited for visualization of the biliary ductal system and surrounding tissue, in particular to evaluate extraluminal structures.

Advantages of MRCP over ERCP include: MRCP has the ability to see extra-luminal findings. MRCP can visualize excluded (obstructed) ducts. MRCP is non-invasive.

Disadvantages of MRCP compared to ERCP include: MRCP does not allow for concurrent therapeutic intervention. MRCP does not actively distend the biliary ductal system with contrast. MRCP has worse spatial resolution compared to ERCP.

Contrast-enhanced MRCP can also be performed with fat-saturated T1-weighted imaging after injection of gadolinium contrast agents that have biliary excretion, such as gadoxetic acid disodium (Eovist, Bayer Healthcare, Germany) and gadobenate dimeglumine (Multihance, Bracco Diagnostics). These agents shorten T1 relaxation, resulting in T1 hyperintense biliary fluid, but require a 20-45 minute delay prior to imaging to allow time for biliary excretion.

How well did you know this?
1
Not at all
2
3
4
5
Perfectly
31
Q

Choledochal Cysts

A

Type I choledochal cyst: Fusiform common bile duct dilation

Type II choledochal cyst: Extrahepatic saccular dilation

Type III choledochal cyst: Dilation of intraduodenal bile duct.

Type IV choledochal cyst: Multiple segments dilated

Type V choledochal cyst: Intrahepatic dilation = Caroli disease

Choledochal cysts are thought to represent a heterogenous group of disease with a common end pathway of intrahepatic or extrahepatic biliary ductal dilation.

The Todani system divides the cysts into types I-V based on their number, distribution, and morphology.

Most choledochal cysts are diagnosed in childhood, but less commonly may be a new diagnosis for an adult. Clinijcally, choledochal cysts can present with nonspecific abdominal pain or may be found incidentally.

Choledochal cysts are often resected due to increased cholangiocarcinoma risk, which can be as high as 25%.

In contrast to biliary hamartomas, choledochal cysts do communicate with the biliary tree.

How well did you know this?
1
Not at all
2
3
4
5
Perfectly
32
Q

Type I Choledochal Cyst

A

A type I choledochal cyst, representing extrahepatic dilation of the common bile duct, is the most common type of extrahepatic cyst.

How well did you know this?
1
Not at all
2
3
4
5
Perfectly
33
Q

Caroli Disease (Type V choledochal cyst)

A

Caroli disease represents saccular dilation of the intrahepatic bile ducts, which may be segmental or diffuse. Caroli disease may be associated with polycystic kidneys.

Caroli syndrome is Caroli disease plus hepatic fibrosis.

The central-dot sign describes the small branches of the portal veink and hepatic artery bridging the dilated bile ducts, which look like a central dot on contrast-enhanced CT.

How well did you know this?
1
Not at all
2
3
4
5
Perfectly
34
Q

Biliary Anatomical Variants

A

Low insertion of cystic duct - With a low insertion of the cystic duct, the surgeon may misidentify the common duct as the cystic duct if the patient undergoes choleycystectomy, possibly leading to inadvertent common duct litigation.

Aberrant right posterior duct - An aberrant right posterior duct is only important if the patient is a right hepatic lobe liver donor, as the two right hepatic ducts need to be anastomosed separately in the recipient.

How well did you know this?
1
Not at all
2
3
4
5
Perfectly
35
Q

Acute Choleycystitis

A

Choleycystitis is inflammation and localized infection secondary to obstruction of the gallbladder neck or cystic duct.

Calculous choleycystitis is caused by a gallstone which blocks the cystic duct.

Acalculous choleycystitis is a functional obstruction of the cystic duct without a culprit stone. It is typically seen in ICU patients.

Acute choleycystitis is typically diagnosed by ultrasound, but similar criteria can be applied to CT: (Gallbladder wall thickening >3 mm (a nonspecific finding). Pericholeycystic fluid or inflammatory changes in the pericholeycystic fat. Gallbladder hyperemia. Gallbladder calculi (although not all gallstones are radiopaque; ultrasound is more sensitive).

Complications of acute choleycystitis include gangrenous choleycystitis, gallbladder perforation, and emphysematous choleycystitis.

Gangrenous choleycystitis is due to increased intraluminal pressure, leading to gallbladder wall ischemia. On imaging, the gallbladder wall thickening may be notably asymmetric and intraluminal membranes may be present. Due to the increased risk of perforation, treatment is emergent choleycystectomy or choleycystostomy.

Acute gallbladder perforation has a very high mortality due to generalized bile peritonitis. Subacute perforation may lead to a pericholeycystic abscess and chronic perforation may cause a choleycystoenteric fistula.

Emphysematous cholecystitis is a severe complication of acute cholecystitis caused by gas-forming bacteria. Gas may be present either within the lumen or the wall of the gallbladder. The typical patient susceptible to emphysematous cholecystitis is an elderly diabetic. Treatment of emphysematous cholecystitis is most often emergent cholecystectomy or cholecystostomy, although treatment can be conservative in patients with a very high surgicla risk.

How well did you know this?
1
Not at all
2
3
4
5
Perfectly
36
Q

Porcelain Gallbladder

A

Porcelain gallbladder describes a peripherally calcified gallbladder wall, thought to be a sequela of chronic cholecystitis.

Porcelain gallbladder is associated with a (somewhat controversial) increased risk of gallbladder carcinoma. Typically, a porcelain gallbladder is an indication for non-emergent cholecystectomy.

How well did you know this?
1
Not at all
2
3
4
5
Perfectly
37
Q

Ascending Cholangitis

A

Obstruction of the biliary tree, most commonly due to choledocholithiasis, may cause ascending cholangitis, which presents with the clinical triad of fever, abdominal pain, and jaundice (Charcot’s triad).

On imaging, the key finding is hyperenhancement and thickening of the walls of the bile ducts, often with a common bile duct stone present. On ultrasound, debris within the biliary system may be apparent.

Initial treatment is antibiotics and fluid resuscitation. Endoscopic biliary intervention may be necessary if the patient does not respond to conservative management.

How well did you know this?
1
Not at all
2
3
4
5
Perfectly
38
Q

Primary Sclerosing Cholangitis

A

Primary sclerosisng cholangitis (PSC) is idiopathic inflammation and destruction of bile ducts

PSC is associated with ulcerative colitis (UC) and is more common in males. (Most (75%) patients with PSC have UC, while only a few (4-5%) of patients with UC have PSC)

Biliary imaging shows a characteristic beaded, irregular appearance of the common bile duct and intrahepatic bile ducts.

PSC appears similar to HIV-cholangiopathy, although cholangitis in HIV patients is more commonly associated with papillary stenosis.

Long-term complications of PSC include cirrhosis, cholangiocarcinoma, and recurrent biliary infections. Cross-sectional imgin is better at evaluating for these complications compared to ERCP.

How well did you know this?
1
Not at all
2
3
4
5
Perfectly
39
Q

Primary Biliary Cirrhosis

A

Primary biliary cirrhosis (PBC is inflammation and destruction of smaller bile ducts compared to PSC. PBC affects middle-aged women and often initially presents with pruritus.

Similar to PSC, chornic PBC can lead to hepatic cirrhosis.

How well did you know this?
1
Not at all
2
3
4
5
Perfectly
40
Q

AIDS cholangitis (AIDS cholangiopathy)

A

Patients with acquired immunodeficiency syndrome are susceptible to biliary infection with Cryptosporidium and CMV, which clinically present with right upper quadrant pain, fever, and elevated LFTs.

The imagin of AIDS cholangitis appears nearly identical to primary sclerosing cholangitis, with multiple strictures and a beaded appearance of the bile ducts. A distinguishing feature of AIDS cholangitis is papillary stenosis, which is not typically seen in PSC.

How well did you know this?
1
Not at all
2
3
4
5
Perfectly
41
Q

Recurrent Pyogenic Cholangitis (Oriental cholangiohepatitis)

A

Recurrent pyogenic cholonagitis, also known as Oriental cholangiohepatitis, is thought to be caused by the parasite Clonorchis sinensis, which leads to pigment stone formation, biliary stasis, and cholangitis. Nutritional deficiency may also play a role. The disease typically affects patients indigenous to Southeast Asia. Clinically, patients present with recurrent jaundice and fevers.

Recurrent pyogenic cholangitis features an imaging triad of: 1) Pneumobilia. 2) Lamellated bile duct filling defects. 3) Intrahepatic and extrahepatic bile duct dilation and strictures.

Patients with recurrent pyogenic cholangitis have an increased risk of cholangiocarcinoma.

How well did you know this?
1
Not at all
2
3
4
5
Perfectly
42
Q

Biliary Cystadenoma

A

Biliary cystadenoma is a benign neoplasm, occuring predominantly in middle-aged women. Biliary cystadenoma may be quite large at presentation and cause nonspecific symptoms such as abdominal pain, nausea, vomiting, and obstructive jaundice.

Biliary cystadenoma does not communicate with the biliary system.

On imaging, biliary cystadenoma appears as a large, multiloculated, cystic mass. The presence of septations distinguishes cystadenoma from a simple cyst. The septations may mimic an echinococcal cyst. In contrast to hepatic abscess or necrotic metastasis, a thick enhancing wall is not a feature of cystadenoma.

Although benign, cystadenoma may uncommonly recur after resection.

Malignant degeneration to biliary cystadenocarcinoma has been reported but is rare. The presence of a large solid component or thick calcification should raise concern for cystadenocarcinoma.

How well did you know this?
1
Not at all
2
3
4
5
Perfectly
43
Q

Cholangiocarcinoma

A

Cholangiocarcinoma is a highly malignant tumor of the biliary ductal epithelium.

A hilar tumor (at the confluence of the right and left intrahepatic biliary ducts), known as a Klatskin tumor, is the most common form of cholangiocarcinoma. In contrast, peripheral cholangiocarcinoa is rare.

Cholangiocarcinoma tends to obstruct bile ducts and cause intrahepatic ductal dilation. Eventually, the obstruction may lead to lobar atrophy.

Risk factors for development of cholangiocarcinoma include: Choledochal cyst(s). Primary sclerosing cholangitis. Familial adenomatous polyposis syndrome. Clonorchis sinensis infection. Thorium dioxide (alpha-emitter contrast agent), not used since the 1950s. Thorium dioxide is also associated with angiosarcoma and HCC.

On cross-sectional imaging, cholangiocarcinoma typically presents as an intrahepatic mass at the confluence of the central bile ducts (Klatskin tumor), with resultant bile duct dilation and capsular retraction. Tumor fingers often extend into the bile ducts.

How well did you know this?
1
Not at all
2
3
4
5
Perfectly
44
Q

Gallbladder carcinoma

A

Gallbladder carcinoma is rare and is usually due to chronic gallbladder inflammation.

Gallstones and concomitant chronic cholecystitis are typically present. Porcelain gallbladder, a result of chronic cholecystitis, is thought to be a risk factor for gallbladder cancer, although this is controversial.

Gallbladder carcinoma most commonly presents as scirrhous infiltrating mass that invades through the gallbladder wall into the liver. Less commonly, gallbladder carcinoma may appear as a polypoid mass. Very rarely it can present as mural thickening.

Tumor spread is via direct extension into the liver, although lymphatic and hematogenous metastasis are also common.

Prognosis is generally poor, although small polypoid lesions may undergo curative resection.

How well did you know this?
1
Not at all
2
3
4
5
Perfectly
45
Q

Metastasis to the Gallbladder

A

Melanoma has a propensity to metastasize to the gallbladder.

How well did you know this?
1
Not at all
2
3
4
5
Perfectly
46
Q

Overview of Pancreatic Neoplasms

A
How well did you know this?
1
Not at all
2
3
4
5
Perfectly
47
Q

Adenocarcinoma (ductal adenocarcinoma)

A

Pancreatic ductal adenocarcinoma makes up 80-90% of all pancreatic tumors. It is typically seen in patients over age 60, with a slight male predominance. Risk factors include smoking, alcohol, and chronic pancreatitis.

A pancreatic-mass CT includes unenhanced, late arterial phase, and portal venous phase images. The late arterial phase (pancreatic parenchymal phase) has the greatest conspicuity for detecting the hypoenhancing tumor against the background enhancing pancreas.

The most common location of ductal adenocarcinoma is the pancreatic head.

The classic appearance is a hypodense (CT), T1 hypointense (MR), ill-defined, hypovascular mass causing ductal obstruction and atrophy of the pancreatic tail. The double duct sign describes dilation of both the pancreatic duct and the common bile duct.

Since pancreatic adenocarcinoma is almost alsways associated with a dilated pancreatic duct, an alternative diagnosis should be strongly considered if there is a pancreatic mass with no ductal dilation, such as: Autoimmune pancreatitis. Groove Pancreatitis. Cystic pancreatic tumor. Neuroendocrine tumor. Duodenal gastrointestinal stromaltumor (GIST). Peripancreatic lymph node. Pancreatic metastasis (e.g., renal cell, thyroid, or melanoma). Lymphoma.

Conversely, if the double duct sign is present but no mass is visible, one should still be suspicious for pancreatic adenocarcinoma. Approximately 10% of cases will be isoattenuating relative to pancreas in the pancreatic parenchymal (late arterial) phase and thus extremely difficult to directly detect.

Most cancers present at an advanced, unresectable stage. Unresectable tumors show encasement (>180° circumference) of the SMA, extensive venous invasion, or evidence of metastasis.

For lower-stage tumors, complete surgical resection is the only chance for cure. A resectable tumor features no evidence of celiac, SMA, or portal venous invasion. Limited extension to the duodenum, distal stomach, or CBD does not preclude resection, as these structures are resected during the Whipple procedure. Limited venous extension may be resectable.

How well did you know this?
1
Not at all
2
3
4
5
Perfectly
48
Q

Acinar cell carcinoma

A

Acinar cell carcinoma is a rare, aggressive variant of pancreatic adenocarcinoma, exclusively seen in elderly males.

The malignant cells produce a large amoutn of lipase to cause the clinical triad of lipase hypersecretion syndrome: Subcutaneous fat necrosis; bone infarcts causing polyarthralgias; and eosinophilia.

How well did you know this?
1
Not at all
2
3
4
5
Perfectly
49
Q

Cystic Pancreatic Epithelial Neoplasms

A

Serous cystadenoma

Mucinous cystic neoplasm

Solid and papillary epithelial neoplasm (SPEN)

Intraductal papillary mucinous neoplasm (IPMN)

How well did you know this?
1
Not at all
2
3
4
5
Perfectly
50
Q

Serous Cystadenoma

A

Serous cystadenoma is a benign tumor that occurs in elderly women and has been nicknamed the grandmother tumor.

It consists of many small cysts (>6 cysts that are <2 cm) that may have a solid appearance on CT due to apposition of many cyst walls. MRI is useful to show the cystic nature of the lesion.

Serous cystadenoma is hypervascular, unigque among cystic pancreatic tumors.

Unlike adenocarcinoma, serous cystadenoma does not cause pancreatic duct dilation or tail atrophy.

A classic imaging feature is central stellate calcification.

How well did you know this?
1
Not at all
2
3
4
5
Perfectly
51
Q

Mucinous cystic neoplasm

A

Mucinous cystic neoplasm affects middle-aged women and has therefore been nicknamed the mother tumor.

Mucinous cystic neoplasm is benign, but does have malignant potential. Treatment is typically resection due to malignant potential.

The tumor consists of a single or a few large cysts (<6 cysts that are >2 cm) and typically occurs in the pancreatic body and tail.

Mucinous cystic neoplasm has a capsule. The only other pancreatic tumor with a capsule is SPEN.

How well did you know this?
1
Not at all
2
3
4
5
Perfectly
52
Q

Solid and Papillary Epithelial Neoplasm

A

Solid and papillary epithelial neoplasm (SPEN) occurs in young women and children and is nicknamed the daughter tumor. It may be a rare cause of abdominal pain.

It has low malignant potential and is typically resected.

On imaging, SPEN appears as a large mass with heterogenous solid and cystic areas. Hemorrhage is typical. SPEN features a capsule, as does mucinous cystic neoplasm.

How well did you know this?
1
Not at all
2
3
4
5
Perfectly
53
Q

Intraductal Papillary Mucinous Neoplasm (IPMN)

A

Intraductal papillary mucinous neoplasm (IPMN) occurs most commonly in elderly males and is nicknamed the grandfather tumor, although these tumros exhibit the greatest age and sex variability of the cystic pancreatic neoplasms.

IPMN featrues a spectrum of biological behavior from benign to indolent to aggressive carcinoma. IPMNs may arise from the main pancreatic duct or a sidebranch. The main duct IPMNs have greater malignant potential.

The classic appearance on endoscopy is a fish-mouth papilla pouring out mucin. Cross-sectional imaging shows a cystic intrapancreatic lesion in contiguity with the duct or sidebranch. Any nodular or enhancing component should raise concern for malignancy.

The recommended imaging follow-up and criteria for resectability are controversial. Current guidelines published in 2006 recommend following simple pancreatic cysts <1 cm annually by imaging (typically MR). However, up to 40% of elderly males have a pancreatic cyst, suggesting that they may be an acquired condition of aging rather than a premalignancy.

In general, a suspected IPMN is resected if it is >3 cm in size, if there is a mural nodule, or if there is associated dilation of the pancreatic duct to >10 mm.

How well did you know this?
1
Not at all
2
3
4
5
Perfectly
54
Q

Pancreatic Endocrine Neoplasms

A

Pancreatic neuroendocrine tumors may be hyperfunctioning or non-hyperfunctioning.

Hyperfunctioning tumors come to clinical attention due to symptoms of endocrine excess.

Non-hyperfunctioning tumors tend to be larger at diagnosis. These tumors may undergo cystic change and should be considered in the differential of a cystic pancreatic neoplasm. There is often central necrosis and calcification in these large tumors as well.

Pancreatic endocrine tumors tend to be hypervascular and are best seen in the late arterial phase. Most are solid unless large. A hypervascular liver mass with an associated pancreatic mass is most likely a metastatic pancreatic endocrine neoplasm.

How well did you know this?
1
Not at all
2
3
4
5
Perfectly
55
Q

Insulinoma

A

Insulinoma is the most common pancreatic endocrine tumor. Due to symptoms of hypoglycemia, insulinomas tend to present early and have the best prognosis of all neuroendocrine tumors, with only 10% demonstrating malignant behavior.

The Whipple triad describes the clinical symptoms of insulinoma: Hypoglycemia, clincial symptoms of hypoglycemia, and alleviation of symptoms after administration of glucose.

How well did you know this?
1
Not at all
2
3
4
5
Perfectly
56
Q

Gastrinoma

A

Gastrinoma causes hypersecretion of gastric acid resulting in Zollinger-Ellison syndrome. Gastrinoma is the second most common pancreatic tumor. Gastrinoma is associated with multiple endocrine neoplasia (MEN) type 1. When associated with MEN-1, gastrinomas tend ot be multiple and located in the duodenum rather than the pancreas.

The gastrinoma triangle describes the typical location of gastrinomas, in an area bounded by the junction of the cystic duct and CBD, the duodenum inferiorly, and the neck/body of the pancreas medially.

High gastrin levels may cause formation of carcinoid tumors in the stomach, which may regress after the gastrinoma is resected.

How well did you know this?
1
Not at all
2
3
4
5
Perfectly
57
Q

Other pancreatic endocrine tumors

A

Glucagonoma is the third most common pancreatic tumor. Prognosis is poor. VIPoma and somatostatinoma are very rare and also have poor prognosis.

How well did you know this?
1
Not at all
2
3
4
5
Perfectly
58
Q

Normal pancreatic ductal anatomy

A

Normally, the main pancreatic duct drains to the major papilla (the ampulla of Vater) through the duct of Wirsun, while the duct of Santorini drains to the minor papilla.

Mnemonic for normal anatomy: Santorini is superior and drains to small (minor) papilla.

The following anatomy is always constant, regardless of whether an anomaly is present: 1) The common bile duct always drains to the major papilla where it meets the duct of Wirsung. 2) The main pancreatic duct alsways drains the pancreatic tail. 3) The duct of Santorini always drains to the minor papilla.

How well did you know this?
1
Not at all
2
3
4
5
Perfectly
59
Q

Pancreas divisum

A

Pancreas divisum is the most common congenital pancreatic anomaly. It is caused by failure of fusion of ventral and dorsal pancreatic ducts. The ventral duct (Wirsung) only drains a portion of the pancreas while the majority of the pancreatic exocrine gland output is drained through the smaller duct of Santorini into the minor papilla.

Pancreas divisum may be a cause of pancreatitis due to obstruction at the minor papilla from a Santorinicele. A Santorinicele is a focal dilation of the terminal duct of Santorini.

The crossing sign describes the CBD crossing over the main duct to join the duct of Wirsung.

How well did you know this?
1
Not at all
2
3
4
5
Perfectly
60
Q

Annular pancreas

A

Annular pancreas is a rare congenital anomaly where a portion of the pancreas wraps completely around the duodenum, secondary to incomplete rotation of the bentral pancreatic bud.

In an adult, annular pancreas can cause pancreatitis, peptic ulcer disease, and duodenal obstruction. In a neonate, it can cause duodenal obstruction and is in the differential for the double bubble sign.

How well did you know this?
1
Not at all
2
3
4
5
Perfectly
61
Q

Common channel syndrome/ pancreatobiliary maljunction

A

Normally the common bile duct and duct of Wirsung both drain to the major papilla, where there is usually a thin septum separating these two systems.

In common channel syndrome, also known as pancreaticobiliary maljunction, the distal CBD and pancreatic duct are missing the septum, allowing refulcx between the two systems.

Common channel syndrome may be in the spectrum of choledochal cyst disease witht he common channel representing a very mild form of choledochocele. Common channel syndrome may predispose to cholangiocarcinoma, but this is rare and controversial.

How well did you know this?
1
Not at all
2
3
4
5
Perfectly
62
Q

Systemic diseases that affect the pancreas

A

Pancreatic manifestations of von Hippel-Lindau - von Hippel-Lindau is an inherited multisystemic disease with icnreased risk of multiple malignancies and formation of cysts in various organs including the pancreas. Pancreatic neoplasms seen in von Hippel-Lindau include serous cystadenoma and pancreatic neuorendocrine tumors.

Cystic fibrosis (CF) - Cystic fibrosis (CF) is the most common cause of childhood pancreatic atrophy. CF can cause either fatty atrophy of the pancreas or pancreatic cystosis (diffuse replacement of the pancreas with innumerable cysts)

Schwachman-Diamond - Schwachman-Diamond is a rare inherited disorder characterized by diffuse fatty replacement of the pancreas, resultant pancreatic exocrine insufficiency, neutropenia, and bone dysplasia. Schwachman-Diamond is the second most cause of childhood pancreatic atrophy.

Obesity and exogenous steroid use - Both obesity and steroids can cause fatty atrophy of the pancreas.

How well did you know this?
1
Not at all
2
3
4
5
Perfectly
63
Q

Intrapancreatic accessory spleen

A

Intrapancreatic accessory spleen is a gening mimic of a hypervascular pancreatic neoplasm.

On imaging, an intrapancreatic spleen typically is a small (1-3 cm), well-defined mass usually found in the pancreatic tail. It follows the density, signal intensity, and enhancement of the spleen on all CT and MRI sequences.

MRI is usually diagnostic. Either technetium-99m sulfur colloid or technetium-99m RBC scintigraphy can confirm the diagnosis in ambiguous cases.

How well did you know this?
1
Not at all
2
3
4
5
Perfectly
64
Q

Pancreatitis

A

Pancreatitis is inflammation of the pancreas, which may be due to a wide variety of etiologies that share a final common pathway of premature activation of pancreatic enzymes and resultant autodigestion of pancreatic parenchyma.

Pancreatitis may range in severity from mild self-limited disease to necrotizing pancreatitis resulting in multi-organ failure and death.

Imaging of pancreatitis is optimally performed in the pancreatic parenchymal phase (late arterial; ~40 seconds after contrast injection), which is the most sensitive timing to detect subtle areas of decreased enhancement suggestive of necrosis.

CT is key for pancreatitis imaging. In addition to often identifying an etiology of the pancreatitis, CT can grade severity, detect complications, and guide possible percutaneous interventions.

CT imaging is not indicated in patients with clinical diagnosis of mild acute pancreatitis, especially if they are improving. CT imaging may be negative or show a mildly edematous pancreas in these cases.

How well did you know this?
1
Not at all
2
3
4
5
Perfectly
65
Q

Acute Pancreatitis

A

Acute pancreatitis is most commonly caused by alcohol or an obstructing gallstone.

Acute pancreatitis can be classified either with the Balthazar grading system or by the CT severity index.

Balthazar grading system: A. Normal-appearing pancreas, B. Focal or diffuse pancreatic enlargement, C. Mild peripancreatic inflammatory changes, D. Single fluid collection, E. Two or more fluid collections. (0% mortality, 4% morbidity for grades A, B, and C; 14% mortality, 54% morbidity for grades D and E (a fluid collection is a poor prognostic indicator).

CT severity index (CTSI) integrates the Balthazar grading system wiht the degree of necrosis: Assigns 0-4 points for Balthazar A-E, with 0 points for Balthazar A and 4 points for Balthazar E. Adds 0-6 points for necrosis to create a total score from 0-10. 0 points: 0% necrosis; 2 points: <30% necrosis, 4 points: 30-50% necrosis, 6 points: >50% necrosis. CTSI 0-3: 3% mortality, 8% morbidity; CTSI 7-10: 17% mortality, 92% morbidity

How well did you know this?
1
Not at all
2
3
4
5
Perfectly
66
Q

Acute pancreatitis pancreatic and peripancreatic complications

A

Pancreatic necrosis: On imaging, pancreatic necrosis appears as a focal or diffuse area of nonenhancing pancreatic parenchyma. Evaluation of necrosis is best performed 48-72 hours after onset of acute pancreatitis. Late arterial phase imaging has the highest sensitivity for detecting pancreatic necrosis. Patients with pancreatic necrosis are at increased risk for infection and severe morbidity.

Fluid collections: Peripancreatic fluid may resolve or may evolve either into peripancreatic abscess or pseudocyst

Pseudocyst: A pancreatic pseudocyst is a collection of pancreatic enzymes and fluid enclosed by a fibrous wall lacking an epithelial lining. The fibrous wall usually takes about 4-6 weeks to mature.

Pancreatic abscess: Pancreatic abscess is a purulent collection featuring thicker, more irregular walls compared to a pseudocyst. Gas locules may be present within the abscess.

How well did you know this?
1
Not at all
2
3
4
5
Perfectly
67
Q

Acute pancreatitis extrapancreatic complications

A

Extrapancreatic pseudocyst may occur nearly anywhere below the diaphragm and should always be considered in the differential of a cystic structure in a patient with history of pancreatitis. In particular, an intrasplenic pseudocyst may lead to intrasplenic hemorrhage.

Perihilar renal inflammation, which may lead to venous compression or thrombosis.

Bowel involvement, especially of the transverse colon.

How well did you know this?
1
Not at all
2
3
4
5
Perfectly
68
Q

Acute pancreatitis secondary inflammation of adjacent vessels

A

Secondary inflammation of adjacent vessels can cause vascular complications:

Arterial bleeding, most commonly due to erosion into the splenic artery.

Pseudoaneurysm, most commonly of the splenic artery.

Venouse thrombosis, most commonly splenic vein thrombosis, which may lead to portal hypertension.

How well did you know this?
1
Not at all
2
3
4
5
Perfectly
69
Q

Chronic pancreatitis

A

Chronic pancreatitis, most commonly from long-term alcohol abuse, causes irreversible pancreatic damage. (A much less common cause of chronic pancreatitis is pancreas divisum)

Calcifications in the distribution of the pancreatic duct are pathognomonic for chronic pancreatitis.

How well did you know this?
1
Not at all
2
3
4
5
Perfectly
70
Q

Autoimmune pancreatitis

A

Autoimmune pancreatitis is caused by inflammatory lymphoplasmacytic infiltrate. It is associated with Sjogren syndrome and causes elevated serum IgG-4 levels.

The typical imaging appearance of autoimmune pancreatitis is diffuse “sausage-shaged” enlargement of the entire pancreas; however a focal or segmental form may mimic a pancreatic mass.

Treatment is with steroids, which can lead to a complete resolution.

How well did you know this?
1
Not at all
2
3
4
5
Perfectly
71
Q

Groove pancreatitis

A

Groove pancreatitis is an uncommon form of focal pancreatitis fo the groove between the head of the pancreas, duodenum, and common bile duct. Groove pancreatitis usually affects young men who are heavy drinkers.

The histopathologic hallmark is fibrosis in the pancreaticoduodenal groove. Chronic inflammation of the duodenum can cause varying levels of duodenal stenosis or cystic change of the duodenal wall. Imaging reflects these findings, with duodenal thickening and cystic change often apparent. The cystic change is best appreciated on MR.

The main differential consideration is adenocarcinoma of the head of the pancreas.

How well did you know this?
1
Not at all
2
3
4
5
Perfectly
72
Q

Splenule

A

Also called an accessory spleen, a spenule is a focus of normal splenic tissue separate from the main body of the spleen, due to embyrologic failure of fusion of the splenic anlage. The most common location is the splenic hilum.

Although usually an incidental finding, the presence of a splenule does have significance in certain clinical settings. For instance, splenectomy for consumptive thrombocytopenia may not be curative if there is sufficient unresected accessory splenic tissue present. A splenule may be mistaken for a lymph node or mass when in an unusual location. As previously discussed, an intrapancreatic splenule may be mistaked for a hypervascular pancreatic mass.

A splenule should follow splenic tissue on all MRI sequences. If in doubt, a Tc-99m sulfur colloid scan or a heat-damaged Tc-99m RBC scan can be confirmatory.

How well did you know this?
1
Not at all
2
3
4
5
Perfectly
73
Q

Polysplenia syndrome

A

Polysplenia syndrome is a spectrum of anatomic disorders characterized by some degree of visceral heterotaxia in addition to multiple discrete foci of splenic tissue. Multiple spleens may be on the right or left, but are always on the same side as the stomach.

Polysplenia is usually associated with severe congenital cardiac anomalies. Most patients die in early childhood, but a few may have only minor cardiac defects and may be incidentally discovered as adults.

Polysplenia is associated with venous anomalies including interruption of the IVC with azygos or hemiazygos continuation. A less common association is a preduodenal protal vein.

How well did you know this?
1
Not at all
2
3
4
5
Perfectly
74
Q

Wandering spleen

A

A wandering spleen is a normal spleen with abnormal laxity or absence of its fixed ligamentous attachments

Wandering spleen may present clinically as an abdominal mass or may cause acute abdominal pain secondary to torsion.

How well did you know this?
1
Not at all
2
3
4
5
Perfectly
75
Q

Hemangioma

A

Hemangioma is the most common benign splenic neoplasm. Hemangioma may be solitary or multiple, and lesions tend to be small.

Splenic hemangiomas are associated with Kasabach-Merritt syndrome (anemia, thrombocytopenia, and consumptive coagulopathy) and Klippel-Trenaury-Weber syndrome (cutaneous hemangiomas, varicose veins, and extremity hypertrophy). These visceral hemangiomatosis syndromes are usually associated with phleboliths.

On CT, hemangiomas are typically iso- or hypoattenuating pre-contrast and hyperenhancing. On MR, hemangiomas are typically hyperintense on T2-weighted images and may enhance peripherally or homogenously. However, the classic pattern of discontinous nodular enhancement seen in hepatic hemangiomas is uncommon.

Nuclear medicine scintigraphy with Tc-99m labeled red blood cells would show increased activity within the lesion on delayed images. In contrast, Tc-99m sulfur colloid scanning may show either increased or decreased activity.

How well did you know this?
1
Not at all
2
3
4
5
Perfectly
76
Q

Hamartoma

A

Splenic hamartoma is a rare, benign lesion composed of malformed red pulp elements. It may be associated with tuberous sclerosis.

Splenic hamartoma is typically a well-circumscribed, iso- or hypoattenuating mass on unenhanced CT that enhances heterogenously after contrast administration. On MR, a hamartoma is iso- to slightly hyperintense on T2-weighted images, featuring heterogenous early enhancement and relatively homogenous delayed enhancement.

How well did you know this?
1
Not at all
2
3
4
5
Perfectly
77
Q

Congenital true (epithelial) cyst

A

A congenital true cyst is defined as having an epithelial lining. Interestingly, a splenic epithelial cyst may cause elevation of tumor markers including CA19-9, CA125, and CEA, despite its completely benign nature.

Unlike a post-traumatic pseudocyst, a true cyst may have septations, but mural calcification is uncommon.

How well did you know this?
1
Not at all
2
3
4
5
Perfectly
78
Q

Post-traumatic pseudocyst

A

A post-traumatic psuedocyst is the end result of evolution of a splenic hematoma.

Unlike a true (epithelial) splenic cyst, the periphery of a pseudocyst is not cellular but made of fibrotic tissue.

On imaging, a post-traumatic pseudocyst appears as a well-circumscribed, fluid-density lesion, with no peripheral enchancement.

In contrast to a true cyst, septations are uncommon but there may be mural calcification.

How well did you know this?
1
Not at all
2
3
4
5
Perfectly
79
Q

Intrasplenic pancreatic pseudocyst

A

A post-pancreatitis pseudocyst involving the tail of the pancreas may extend into the spleen. There is almost always a history of pancreatitis.

Unlike a true congenital cyst, an epithelial lining is lacking and histology more closely resembles a post-traumatic pseudocyst.

Splenic rupture has been reported in some cases of intrasplenic post-pancreatitis pseudocysts.

80
Q

Lymphangioma

A

Splenic lymphangioma is a rare, benign neoplasm usually diagnosed in childhood, which may be solitary or multiple.

Lymphangioma features a classic imaging appearance of a multilocular cystic structure with thin septations. Post-contrast images may show septal enhancement.

81
Q

Sarcoidosis

A

Sarcoidosis is a systemic disease of unknown etiology characterized histologically by multiple nodules composed of noncaseating granulomas.

When sarcoidosis involves the spleen, splenomegaly is the most common presentation, often associated with hepatomegaly and lymphadenopathy.

Less commonly, sarcoidosis may involve the spleen in a multinodular pattern with numerous hypoattenuating 1-3 cm lesions

82
Q

Inflammatory pseudotumor

A

Splenic inflammatory pseudotumor is a rare focal collection of immune cells and associated inflammatory exudate, of unclear etiology. Patients often have constitutional symptoms including fever and malaise.

Inflammatory pseudotumor has a variable and nonspecific imaging appearance, but a typical presentation is of a well-circumscribed, heterogenously enhancing mass.

83
Q

Pyogenic Abscess

A

Splenic bacterial abscesses are uncommon and usually seen in immunocompromised patients. A solitary abscess is much more likely to be bacterial. In contrast, multifocal small abscesses are more likely to be fungal.

On CT, a bacterial abscess usually has an irregular, enhancing wall. Gas is not usually seen, but is highly specific for a bacterial abscess when present.

A characteristic ultrasound finding is the wheel within a wheel or bull’s-eye appearance, which describes concentric hyperechoic and hypoechoic rings surrouding the abscess.

Treatment is CT- or ultrasound-guided percutaneous drainage in addition to antibiotics.

84
Q

Fungal abscess

A

Splenic fungal abscesses are typically multiple and small, usually <1 cm in size. Almost all patients with splenic fungal abscesses are immunocompromised.

The most causative agents include Candida, Aspergillus, and Cryptococcus, which all appear as multiple tiny hypoattenuating foci on CT.

Splenic Pneumocystis jiroveci (formerly known as Pneuocystis carinii) infection is rare, almost always seen in advanced AIDS, and has a classic appearance of multiple calcified splenic lesions.

85
Q

Echinococcal cyst

A

Splenic inolvement of Echinococcus granulosus infection is unusual, seen in only 1 to 3% of echinococcal infections, and is almost always associated with infection of other organs as well.

An echinococcal cyst is a true cyst with a cellular lining. As with echinococcal abscess elsewhere, characteristic imaging findings are a cystic lesion with internal undulating membrane and daughter cysts.

86
Q

Splenic lymphoma

A

Splenic lymphoma is the most common splenic malignancy.

Primary splenic lymphoma is rare, accounting for less than 1% of all cases of lymphoma, and usually presents as a solitary hypovascular mass. In contrast to splenic hamartoma or metastatic disease, primary splenic lymphoma may extend beyond the splenic capsule and involve adjacent organs.

Secondary splenic involvement of systemic lymphoma is much more common. Four imging presentations have been described, depnding on the size of the lymphomatous masses: Miliary masses in which discrete tiny masses may be difficult to see. Multiple small to moderate sized masses. One large mass. Splenomegaly without discrete mass.

Conspicuity is highest on post-contrast T1-weighted images, where the involved portion of the spleen tends to be hypoenhancing.

On ultrasound, lymphoma may appear cystic but color Doppler shows internal flow.

87
Q

Splenic metastasis

A

Splenic metastases may be solitary of multiple, cystic or soid. However, metastases to the spleen are rare, occuring in 2-9% of cancer patients. Most patients with splenic metastases already have known widespread disease (Isolated splenic metastases are seen in only 5% of patients with metastatic involvement of the spleen.)

Several theories for the low rate of splenic metasteses have been proposed, which is thought to be due to the antineoplastic properties of lymphoid-rich splenic tissue and lack of afferent lymphatics to bring tumor cells into the spleen.

The most common primary tumors known to metastasize to the spleen include breast, lung, ovarian, and melanoma. Ovarian cancer and melanoma typically cause cystic metastasis.

Calcification is rare unless the primary is a mucinous adenocarcinoma.

88
Q

Angiosarcoma

A

Angiosarcoma is a rare, extremely aggressive malignancy, with 20% 6-month survival.

Unlike hepatic angiosarcoma, the association with Thorotrast, vinyl cholride, and arsenic have not been well established.

Angiosarcoma usually presents as enlarged, heterogenous mass that may completely replace the normal spleen. Enhancement is variable and heterogenous.

89
Q

Splenic infarct

A

Splenic infarcts are most commonly due to emboli (in older patients) and thrombosis (in younger patients with hematologic disease)

Splenic infarct classically manifests as a wedge-shaped peripheral region of nonenhancement, but a more heterogenous, mass-like appearance can also be seen.

On MR, the affected regions may be T1 hyperintense if acute and hemorrhagic, while chronic infarcts are T1 hypointense and T2 hyperintense.

Lack of enhancement of the entire spleen should raise the concern for complete infarction, possibly due to a wandering spleen with torsion.

90
Q

Gamna-gandy bodies

A

Gamna-Gandy bodies are multiple tiny foci of hemosiderin deposition resulting from portal hypertension.

Usually, other signs of portal hypertension and cirrhosis are apparent on imaging, such as splenomegaly, varices, recanalized umbilical vein, ascites, and nodular liver contour.

The hemosiderin deposits demonstrate low signal on all sequences. On in- and out-of-phase gradient-echo sequences, the longer TE of the in-phase images demonstrate blooming (relatively decreased signal on in-phase images), due to longer dephasing time and exaggeration of T2* effecct.

91
Q

Gaucher disease

A

Gaucher disease is an autosomal recessive deficiency of glucocerebrosidase, leading to accumulation of glucocerbrosides in the reticuloendothelial system.

Splenic manifestations of Gaucher disease include splenomegaly (almost always) and multiple splenic nodules (seen in one third of Gaucher patients).

Associated bony findings seen in Gaucher disease include the characteristic Erlenmeyer flask deformity of the distal femurs, femoral head avascular necrosis, and H-shaped vertebral bodies from endplate avascular necrosis.

92
Q

Overview of splenic trauma

A

The spleen is the most commonly injured abdominal organ from blunt trauma.

Splenectomy dramatically increases the risk of subsequent sepsis, which has driven the trend towards splenic preservation and conservative management of splenic trauma.

A splenic hematoma is a focal collection of blood (hypoattenuating relative to enhanced spleen and hyperattenuating relative to unenhanced spleen), most commonly subcapsular. Less commonly, a hematoma may be intraparenchymal, where it may assume an irregular shape.

A splenic laceration can only be well seen on a contrast-enhanced study, where it appears as a linear or branching area of decreased attenuation.

Active contrast extravasation due to vessel injury appears as an area of increased attenuation (initially iso-enhancing to the arterial blood pool), which further increases in size on delayed scanning due to ongoing bleeding.

In contrast to active extravasation, pseudoaneurysm and arteriovenous fistula are contained vascular injuries that also initially appear as a well-circumscribed area of increased attenuation, but do not increase on delayed scanning.

93
Q

American Association for the Surgery of Trauma (AAST) grading scale for splenic trauma

A

The American Association for the Surgery of Trauma (AAST) splenic injury grading scale is based on the extent of injury seen at laparotomy, and is therefore limited in practicality as many splenic injuries are now managed conservatively.

An additional limitation of the AAST grading scale is the exclusion of vascular injury.

94
Q

MDCT- based splenic injury grading system

A

The most widely used CT grading system is the MDCT-based splenic injury grading system, which is similar to the AAST grading scale but incorporates vascular injury.

MDCT grade I: Small (< 1cm) subcapsular hematoma, laceration, or parenchymal hematoma.

MDCT grade II: Medium (> 1 and < 3 cm) subcapsular hematoma, laceration, or parenchymal hematoma.

MDCT grade III: Splenic capsular disruption; or large (> 3 cm) laceeration or parenchymal hematomas.

MDCT grade IVA: Active extravasation, vascular injury (pseudoaneurysm or arteriovenous fistula), or shattered spleen.

MDCT grade IVB: Active intraperitoneal bleeding.

95
Q

Esophagus Anatomy

A

Nasopharynx: Extends from the base of the skull to the soft palate.

Oropharynx: Located behind the mouth and extends from the uvula to the hyoid bone.

Hypopharynx: Extends from the hyoid bone to the cricopharyngeus muscle, which is located at the lower end of the cricoid cartilage.

Esophagus: The cricopharyngeus muscle, located at C5-6 is the upper esophageal sphincter and demarcates the transition between the pharynx superiorly and the cervical esophagus. The esophagus extends from the neck to the gastroesophageal junction. The distal esophagus passes throught the diaphragmatic hiatus at approximately T10. The three anatomic rings of the distal esophagus are the A (muscular), B (mucosal) and C (diaphragmatic impression) rings.

96
Q

Esophageal web

A

An esophageal web is a thin anterior infolding/indentation of the upper esophagus, which is usually asymptomatic but may be a cause of dysphagia. There is a controversial association with anemia (Plummer-Vinson syndrome) and upper esophageal carcinoma.

97
Q

Schatzki ring

A

A Schatzki ring is a focal narrowing of the B (mucosal) ring of the distal esophagus, causing intermittent dysphagia.

A true Schatzki ring requires clinical symptoms of dysphagia in addition to esophageal narrowing seen on imaging. (Asymptomatic narrowing of the B ring is referred to as a lower esophageal ring.)

An upper GI study is more sensitive than endoscopy. The key imaging feature is focal circumferential constriction near the gastroesophageal junction, almost always associated with a hiatal hernia. (On an upper GI study, most symptomatic rings do not allow passage of a 12 mm tablet.)

The differential of circumferential esophageal constrictions includes: focal stricture, muscular esophageal ring above the GE junction (also known as an A ring), esophageal cancer, esophageal web (rarely circumferential, usually in cervical esophagus).

98
Q

Reflux (peptic) esophagitis

A

Reflux (peptic) esophagitis is caused by exposure of the esophageal mucosa to acidic gastic secretions, which leads to distal ulcerations and eventual stricture.

Peptic esophagitis is most commonly caused by gastroesophageal reflux, but is also seen in: Zollinger-Ellison, due to increased acid production. Scleroderma, due to gastroesophageal sphincter fibrosis and resultant incompetence.

Reflux esophagitis appears as thickened distal esophageal folds.

Chronic esophagitis and scarring develops after prolonged exposure to acid, which causes a smoothly tapered stricture above the GE junction.

99
Q

Barrett esophagus

A

An important long-term sequela of peptic esophagitis is Barrett esophagus, which is metaplasia of normal squamous epithelium to gastric-type adenomatous mucosa. Barrett esophagus is a precursor lesion to esophageal carcinoma.

Nearly 10% of patients with reflux esophagitis may have some adenomatous metaplasia.

On imaging, Barrett demonstrates a featureless distal esophagus, with signs of active reflux esophagitis (mucosal granularity and superficial erosions) more proximally.

Barrett esophagus is often associated with esophageal stricture, which is abnormally high in location compared to a peptic stricture.

100
Q

Infectious esophagitis

A

Although radiographic distinction between types of infections has been described, endoscopy and biopsy are typically performed in clinical practice.

Esophageal candidiasis can present as a spectrum from scattered plaque-like lesions in mild disease to very shaggy esophagus in severe cases.

Herpes esophagitis typically causes discrete small ulcerations scattered randomely throughout esophagus.

CMV/HIV esophagitis characteristically causes a large, flat, ovoid ulcer.

101
Q

Medication Esophagitis

A

Medication-induced esophagitis typically causes an ulcer at the level of the aortic arch or distal esophagus, which are areas of relative narrowing that may predispose to temporary hold-ups in passage.

102
Q

Crohn Esophagitis

A

Crohn esophagitis is very rare and is usually seen in the setting of severe disease in the small bowel and colon.

Apthous ulcers (discrete ulcers surrounded by mounds of edema) may become confluent.

103
Q

Peptic stricture

A

As previously discussed, a peptic stricture is secondary to chronic reflux.

Peptic strictures are located distally, usually just above the GE junction.

A peptic stricture may be focal or involve a longer segment of esophagus.

Fibrosis can cause esophageal shortening, leading to a hiatal hernia as the stomach is pulled into the thorax.

104
Q

Barrett esophagus stricture

A

A Barrett stricture typically occurs in the mid-esophagus, above the metaplastic adenomatous transition. Barrett strictures occur higher than peptic strictures because adenomatous tissue is acid-resistant and therefore unaffected by gastric secretions.

105
Q

Malignant stricture (due to esophageal carcinoma)

A

Key imaging finding is shouldered margins, which suggests circumferential luminal narrowing by a mass.

106
Q

Caustic stricture/nasogastric (NG) tube stricture

A

Both caustic strictures and strictures secondary to nasogastric tube placement are typically long, smooth, and narrow.

Strictures develop 1-3 months after the caustic ingestion or NG tube placement.

Caustic strictures are associated with an increased risk of cancer, with a long lag time of up to 20 years after the initial insult. Caustic strictures are usually longer than peptic strictures.

107
Q

Radiation stricture

A

Radiation strictures are long, smooth and narrow, similar to caustic strictures. However, in contrast to strictures from an NG tube, caustic ingestion, and reflux, radiation strictures usually spare the GE junction.

It generally requires more than 50 Gy of radiation to cause an esophageal stricture.

Acute radiation esophagitis occurs 1-4 weeks after radiation therapy. Radiation strictures develop later, occuring 4-8 months after radiation.

108
Q

Evaluation of Esophageal Masses

A

Masses arising from the mucosa, submucosa, and extrinsic to the esophagus produce characteristic effects on the esophagus, which are usually able to be seen on imaging.

109
Q

Mesenchymal esophageal tumor

A

Benign mesenchymal tumors are the most common submucosal tumors and include gastrointestinal stromal tumor (GIST), leiomyoma, lipoma, hemangioma, and others. The classification varies in the literature, with both GIST and leiomyoma described as the most common. On a barium swallow, a mesenchymal tumor typically appears as smooth, round, submucosal filling defect.

110
Q

Adenoma esophageal mass

A

An esophageal adenoma is a benign mucosal lesion with malignant potential, usually arising within Barrett esophagus. Most are <1.5 cm in size and resected at endoscopy.

111
Q

Inflammatory Polyp (esophagus)

A

An inflammatory polyp is a non-neoplastic, enlarged gastric fold that protrudes up into the lower esophagus. Inflammatory polyps are almost always associated with reflux and always contiguous with a gastric fold. They are mucosal in location.

112
Q

Fibrovascular Polyp (Esophagus)

A

A fibrovascular polyp is apedunculated mass composed of mesenchymal elements with a significant fatty component. In contrast to an esophageal adenoma, there is no malignant potential. Fibrovascular polyps usually occur in the cervical esophagus. The clnical presentation can be dramatic, with regurgitation of a fleshy mass.

CT is usually diagnostic, demonstrating intra-lesional fatty component.

113
Q

Esophageal Varices

A

Esophageal varices are most commonly due to portal hypertension. Varices can usually be distinguished from a solid mass since varices change in size and shape with peristalsis. However, thrombosed varices may mimic a tumor.

Uphill varices, due to portal hypertension, affect the distal esophagus. (Blood flows “uphill” from the portal vein -> left gastric (coronary vein) -> periesophageal venous plexus -> azygous/hemiazygous collaterals -> SVC.

Downhill varices are much less common, are caused by superior vena cava obstruction, and usually affect the proximal esophagus. (Enlarged collateral vessels include the supreme intercostal veins (drain the first intercostal space), bronchial veins, and inferior thyroidal veins.

114
Q

Esophageal foreing body

A

A radiopaque esophageal foreign body is best visualized with a lateral radiograph or CT.

Bony foreign objects usually get stuck in the cervical esophagus.

Meat impaction usually occurs at the gastroesophageal junction. There is a risk of esophageal perforation from transmural ischemia if the food bolus is impacted for >24 hours. Most cases of food bolus impaction are treated with endoscopic removal of the impacted food. Historically, meat impaction was treated with effervescent granules and meat tenderizer, but this technique is no longer commonly performed.

115
Q

Foregut duplication cysts (Esophagus)

A

Esophageal duplication cyst is lined with squamous epithelium, has a smooth muscle wall, and is usually in the posterior mediastinum. It may be either extrinsic to the esophagus or submucosal; the latter is impossible to differentiate from a leiomyoma by esophagram.

Bronchogenic cyst is lined by respiratory epithelium. It is generally indistinguishable from an esophageal duplication cyst on esophagram and CT.

Neurenteric cyst is associated with vertebral body anomalies.

116
Q

Esophageal carcinoma

A

Esophageal carcinoma has a broad range of appearances. Early esophageal cancer may be apparent on barium swallow as a plaque-like lesion, polypoid lesion, or focal irregularity of the esophagea wall. A classic appearance of advanced esophageal carcinoma is a mass causing stricture with a “shouldered” edge and irregular contour. The uncommon varicoid appearance can be initially confused with varices, but the tumor does not chage shape with peristaltic waves as varices typically do.

Esophageal carcinoma may be squamous cell carcinoma (SCC) or adenocarcinoma, which cannot be reliably differentiated on barium studies. SCC tends to involve the upper or mid-esophagus and adenocarcinoma typically involves the distal esophagus and may extend into the stomach.

Squamous cell carcinoma (SCC) is most commonly due to smoking and alcohol. Less common risk factors include celiac disease, Plummer-Vinson, achalasia, and human papilloma virus (which more commonly cuases laryngeal squamous cell carcinoma).

Adenocarcinoma is due to chronic reflux, arising from Barrett esophagus distally. Its incidence has been rising in recent years.

117
Q

Esophageal Metastasis

A

Direct invasion of the esophagus is most commonly from gastric, lung, or breast primaries. Hematogenous spread is very rare.

Most often, mediastinal lymph node metastasis will be prominent. The mid-esophagus is most commonly affected due to its proximity to mediastinal lymph nodes.

118
Q

Esophageal lymphoma

A

Esophageal lymphoma is often indistinguishable from primary esophageal cancer.

119
Q

Esophageal Malignant GIST

A

Malignant GIST tends to be bulkier and more irregular than the benign variant.

120
Q

Esophageal contraction waves

A

A primary contraction wave is a normal, physiologic wave initiated by a swallow.

A secondary contraction wave is a normal, physiologic wave initiated by a bolus in the esophagus.

A tertiary wave is a nonpropulsive contraction that does not result in esophageal clearing. Tertiary contractions are seen more commonly in the elderly. They are not normal, but are also not thought to be clinically significant when seen.

121
Q

Esophageal Achalasia

A

Achalasia is a motility disorder of the distal esophagus, which is unable to relax due to an abnormality of myenteric ganglia in the Auerbach plexus. Vigorous achalasia is a less severe form of achalasia consisting of repetitive nonpropulsive contractions.

Chagas disease causes a secondary achalasia that is indistinguishable radiographically from primary achalasia.

Potential complications of chronic achalasia include esophageal cancer, which has a lag period of at least 20 years, and candidal infection from stasis.

The classic imaging appearance of achalasia is a massively dilated esophagus with a bird’s beak stricture near the gastroesophageal junction.

Surgical treatment of achalasia is the Heller myotomy, which is an incision of the lower esophageal muscle fibers.

Pseudoachalasia is caused by an obstructing gastroesophageal junction cancer. In achalasia, there is transient relaxation of the stricture when the patient stands. In pseudoachalasia, however, the fixed obstruction does not relax with standing.

122
Q

Diffuse esophageal spasm (corkscrew esophagus; shish kebab esophagus)

A

Diffuse esophageal spasm is a clinical syndrome of chest pain or dysphagia caused by repetitive, nonpropulsive esophageal contractions. The nonpropulsive contractions have a characteristic appearance on barium swallow, leading to the descriptive names of corkscrew esophagus and shish kebab esophagus.

Nutcracker esophagus is a related disorder characterized by high-amplitude contractions on manometry in conjunction with chest pain, with normal radiographic findings.

123
Q

Type of esophageal diverticula

A

Pulsion diverticula are caused by increased esophageal pressure and comprise nearly all diverticula seen in the USA.

Traction diverticula are caused by traction of adjacent structures, typically resulting from tuberculous mediastinal adenopathy. They are rarely seen.

124
Q

Zenker diverticulum

A

Zenker diverticulum is an esophageal diverticulum casued by failure of the cricopharyngeus muscle to relax, leading to elevated hypopharyngeal pressure. Symptoms of a Zenker diverticulum include halitosis, aspiration, and regurgitation of undigested food.

A Zenker diverticulum is posteriorly protruding. As a secondary finding, the cricopharyngeus muscle is usually hypertrophied.

Treatment is with cricopharyngeal myotomy and diverticulopley or diverticulectomy.

A pseudo-Zenker diverticulum is barium trapped in a pharyngeal contraction wave.

125
Q

Killian-Jamieson (KJ) diverticulum

A

A Killian-Jamieson (KJ) diverticulum is located at the Killian-Jamieson space, which is an area of wekness below the attachment of the cricopharyngeus muscle.

In contrast to Zenker diverticulum, KJ diverticula are more often bilateral.

KJ diverticula protrude anteriorly, best seen on the lateral view.

126
Q

Pseudodiverticulosis

A

Pseudodiverticulosis is the imaging finding of multiple tiny outpouchings into the esophageal lumen caused by dilated submucosal glands from chronic reflux esophagitis.

These submucosal glands are analogous to the Rokitansky-Aschoff sinuses of the gallbladder.

Pseudodiverticulosis is often associated with a smooth stricture in mid/upper esophagus, which may cause symptoms.

Candida is frequently culture, but infection is not believed to be a causal factor.

127
Q

Feline Esophagus

A

Feline esophagus is thought to be a normal variant characterized by multiple transverse esophageal folds.

There is a controversial association with esophagitis, where the incidence of esophagitis may be increased in the presence of feline esophagus.

128
Q

Aberrant right subclavian artery

A

Aberrant right subclavian artery (with a normal left arch) is seen in approximately 1% of patients and is almost always asymptomatic. The aberrant right subclavian artery travels posterior to the esophagus, where it may rarely produce dysphagia.

On an upper GI study, the resultant posterior esophageal indentation is always smooth.

129
Q

Scleroderma

A

Scleroderma is a systemic disease involving excess collagen deposition in multiple tissues.

The esophagus is involved in 80% of patients with scleroderma, producing lack of peristalsis of the distal 2/3 of the esophagus due to smooth muscle atrophy and fibrosis, which leads to marked esophageal dilation.

Secondary candidiasis or aspiration pneumonia can result from prolonged esophageal stasis.

The esophageal dilation is often apparent before the typical skin changes of scleroderma become evident.

130
Q

Esophageal Hiatal Hernia

A

A hiatal hernia is present when gastric fold are seen above the diaphragm. A hiatal hernia may be sliding (most common) or short (secondary to chronic reflux esophagitis)

131
Q

Paraesophageal hernia

A

With a paraesophageal hernia, the GE junction is located normally below the diaphragm, but a portion of the stomach herniates into the thorax through the esophageal hiatus.

Paraesophageal hernia is more prone to strangulation than HH. Most are surgically repaired.

132
Q

Thickened gastric folds

A

Thickened gastric folds are most commonly due to inflammatory gastritis, which characteristically produces smooth fold thickening.

Nodular fold thickening is suggestive of neoplasm, such as gastric lymphoma or submucosal carcinoma.

133
Q

Helicobacter pyloir gastritis (esophagus)

A

Helicobacter pylori is major cause of gastritis, gastric ulcers, and duodenal ulcers.

134
Q

Zollinger-Ellison (ZE) (Esophagus)

A

Zollinger-Ellison (ZE) is gastrin over-production from a gastrinoma, which is a pancreatic islet cell tumor that has a 50% rate of malignancy. (ZE features elevated gastrin level and a paradoxical increase in gastrin after secretin administration.)

25% of patients with gastrinoma have multiple endocrine neoplasia (MEN) type 1. (MEN-1 consists of parathyroid adenoma, pituitary adenoma, and pancreatic islet cell tumors.)

135
Q

Eosinophilic gastritis

A

Eosinophilic gastritis is characterized by thickened folds in the stomach and small bowel in a patient with a history of allergy.

136
Q

Menetrier disease

A

Menetrier disease is a protein-losing enteropathy that is often a diagnosis of exclusion. It usually affects the proximal stomach and is pathologically characterized by replacement of parietal cells by hyperplastic epithelial cells, leading to achlorhydria.

Menetrier disease has a controversial association with gastric carcinoma.

137
Q

Crohn disease

A

Gastric Crohn disease is almost always associated with small bowel disease. Usually the distal half of the stomach is affected.

The earliest pathologic change is the formation of apthous ulcers.

138
Q

Other causes of thickened gastric folds

A

Gastric varices (from portal hypertension), gastric lymphoma, and submucosal carcinoma are non-inflammatory causes of thickened gastric folds.

139
Q

Hyperplastic polyp (inflammatory polyp) (Gastric)

A

A hyperplastic polyp, also known as an inflammatory polyp, is cystic dilation of a gastric gland that develops in response to chronic inflammation. Hyperplastic polyps are almost always benign, with very rare cases of malignant transformation having been reported.

Fundic gland polyposis syndrome is a variant of familial adenomatous polyposis syndrome that also involves the stomach. In the stomach, most polyps are hyperplastic, but elsewhere in the GI tract the polyps are adenomatous.

140
Q

Hamartomatous polyp (Gastric)

A

Hamartomatous polyps are benign polyps usually associated with syndromes such as Peutz-Jeghers, juvenile polyposis, and Cronkhite-Canada syndromes.

141
Q

Lipoma (Benign gastric mass)

A

A lipoma is a benign, submucosal, mesenchymal neoplasm. At fluoroscopy, a gastric lipoma is indistinguishable from a GIST. Fatty attenuation on CT is diagnostic of a lipoma.

142
Q

Gastrointestinal stromal tumor (GIST)

A

Gastrointestinal stromal tumors (GIST) is the most common submucosal gastric tumor. The tumor arises from the interstitial cells of Cajal, which are pacemaker cells that drive peristalsis. GIST may occur anywhere in the gastrointestinal tract.

GIST may be benign or malignant, with risk for malignancy determined by size and number of mitoses. Regardless of size and number of mitoses, gastric GIST is less likely to be malignant compared to similar-sized GISTs in the duodenum, jejunum/ileum, or rectum. Gastric tumors = 2 cm in size are essentially always benign. Larger tumors carry a risk of malignancy as high as 86% for a gastric GIST > 10 cm with an elevated mitotic rate.

Small gastric GISTs are usually asymptomatic, but may be a cause of melena.

On imaging, a smooth endoluminal surface is characteristic due to its submucosal location. Larger tumors have a tendency to become exophytic, or less commonly to invade the lumen.

The differential diagnosis of a submucosal gastric mass includes mesenchymal tumors (GIST, fibroma, lipoma, neurofibromas, etc.), carcinoid, and ectopic pancreatic rest.

143
Q

Ectopic pancreatic rest

A

An ectopic pancreatic rest is a focus of heterotopic pancreas in the gastric submucosa. The ectopic pancreatic tissue is susceptible to pancreatic diseases, including pancreatitis and carcinoma. On imaging, the classic appearance is an umbilicated submucosal nodule, with the umbilication representing a focus of normal epithelium. The ulceration is not always seen, in which case the imaging is of nonspecific submucosal gastric mass.

144
Q

Gastric cancer

A

Gastric adenocarcinoma may present either as a mass or as a gastric ulcer.

Gastric cancer is generally caused by chronic inflammation, with specific risk factors including: Ingestion of polycyclic hydrocarbons and nitrosamines (from processed meats). Atrophic gastritis. Pernicious anemia. Post-subtotal gastrectomy.

Gastric carcinoma may spread locally from the mucosal surface to the serosa, in which case 90% of patients will have omental involvement from trans-serosal spread.

Lymphatic spread is along lesser curvature -> gastrohepatic ligament and greater curvature.

A Krukenberg tumor is classically described as metastatic spread of gastric carcinoma to the ovary; however, the term has also been used to describe any mucinous metastasis to the ovary.

145
Q

GIST (malignant)

A

Malignant GIST tends to be larger than benign GIST, often reaching sizes of greatr than 10 cm, with central necrosis. Although the tumor begins in the submucosa, it can be difficult to determine the site of origin of large tumors.

146
Q

Lymphoma (gastric)

A

Gastric lymphoma can have a wide variety of presentations. If solitary, lymphoma can mimic gastric carcinoma. To differentiate between lymphoma and gastric carcinoma, the pattern of adenopathy can be helpful. In gastric cancer, adenopathy at or below the level of the renal hila is unusual, but occurs more commonly in patients with lymphoma.

The stomach is a common extranodal site for non-Hodgkin lymphoma.

147
Q

Metastases (Gastric)

A

Metastatic disease to the stomach is rare. Breast, lung, and melanoma are most common.

148
Q

Benign gastric ulcer

A

Although less commonly encountered in the modern era of proton pump inhibitors and Helicobacter pylori treatment, benign gastric ulcers tend to have typical imaging findings: Radiating gastric folds are smooth and symmetric. Ulcer extends beyond the normal contour of the gastric lumen. The Hampton line represents nonulcerated acid-resistant mucosa surrounding the ulcer crater. Most beign ulcers occur along the lesser curvature of the stomach, although benign ulcers associated with aspirin ingestion can occur in the greater curvature and antrum, which are dependent locations.

149
Q

Gastric Carcinoma

A

Gastric carcinoma may present with malignant ulceration, which can usually be distinguished from a benign ulcer by the following features:

  • Asymmetric ulcer crater, with surrounding nodular tissue.
  • Abrubt transition between normal gastric wall and surrounding tissue.
  • Ulcer crater does not project beyond the expected location of gastric wall.
  • The Carman meniscus sign is considered pathognomonic for tumor. It describes the splaying open of a large, flat malignant ulcer when compression is applied.
150
Q

Overview of Gastric bypass surgery

A

In order to evaluate for and accurately describe complications of Roux-en-Y gastric bypass (RYGB) surgery, it is important to be familiar with the procedure and normal postsurgical anatomy.

A small gastric pouch is created with a volume of approximately 15 to 30 cc by excluding the distal stomach from the path of food.

The Roux limb is created by transecting the jejunum approximately 35-45 cm distal to the ligament of Treitz, then bringing it up to be anastomosed to the gastric pouch via a narrow gastrojejunostomy stoma.

The current favored approach for placement of the Roux limb is antecolic (in front of the transverse colon). The Roux limb used to be placed retrocolic, which required the creation of a surgical defect through the transverse retrocolic, which required the creation of a surgical dfect through the transverse mesocolon (mesentery of the transverse colon). A retrocolic Roux limb has a higher risk of a transmesocolic hernia due to the defect in the transverse mesocolon. (Although the antecolic approach is now more commonly performed, there are many patients who have previously undergone a retrocolic approach.)

A distal side-to-side jejunojejunostomy is created to connect the pancreaticobiliary limb to the jejunum.

They RYGB leads to weight loss both from early satiety (due to small size of the gastric pouch) and malabsorption (due to surgical bypass of the proximal jejunum.)

151
Q

Complications of Roux-En-Y surgery

A

Postoperative leak

  • Postoperative leak is usually diagnosed by 10 days after surgery
  • An upper GI study with water-soluble contrast is the study of choice if a leak is suspected.
  • Leaks may arise from the distal esophagus, gastric pouch, or blind-ending jejunal limb. It is rare for a leak to arise from the distal jejunojejunostomy.

Gastrogastic fistula

  • A gastrogastic fistula is a communication between the gastric pouch and the excluded stomach, which may be an early or late complication of the RYGB.
  • A gastrogastric fistula may be a cause of inadequate weight loss or recurrent weight gain.

Small bowel obstruction (SBO)

  • Small bowel obstruction (SBO) in the acute postoperative period is most often due to edema or hematoma at the gastrojejunostomy or jejunojejunostomy. (With a retrocolic Roux limb, edema at the transverse mesocolon defect may also cause obstruction. Treatment is usually conservative, with most cases resolving as the edema and/or hematoma resolves.)
  • A late presentation of small bowel obstruction may be due to internal hernia (more common with laparoscopic surgery) or adhesions (more common with open surgery).

Internal Hernia

  • Laparascopic Roux-en-Y procedures are associated with a higher rate of internal hernias (seen in 2.5% of laparascopic procedures) compared to open procedures (0.5%). Internal hernias can be difficult to diagnose, both clinically and by imaging.
  • Internal hernias usually present within 2 years of bypass and are the most common cause of SBO after a laparoscopic Roux-en-Y
  • Most RYGB-associated internal hernias occur in three characteristic locations.
  • The surgically created defect in the mesentery of the transverse colon is the most common site (the transmesocolic hernia), associated with a retrocolic Roux limb.
  • Less common sites of internal hernia include Peterson’s space (located between the mesentery of the Roux limb and the transverse mesocolon) and the mesenteric defect created by the jejunojejunostomy.
  • Imaging features of internal hernia include swirling of the mesentery, a mushroom shape of the mesentery, and/or the presence of small bowel loops posterior to the superior mesenteric artery.

Stomal stenosis

  • Narrowing of the gastrojejunostomy stoma may occur in up to 10% of patients, leading to dilation of the pouch and distal esophagus. Stomal stenosis is usually treated with endoscopic dilation.
  • Narrowing of the distal jejunojejunostomy is much more rare and usually requires surgery.

Marginal ulcers

  • The jejunal mucosa adjacent to the gastrojejunal anastomosis is susceptible to gastric secretions, which can cause marginal ulcers in up to 3% of patients.
  • A marginal ulcer is diagnosed by upper GI as a thickening and small outpouching of a gastric fold.
  • Treatment is conservative.
152
Q

Small Bowel Anatomy

A

The wall of the small intestine is made of four layers, from outside in: Serosa, Muscularis (thin longitudinal and thick circumferential smooth muscle). Submucosa, Mucosa (consists of intestinal villi, circular folds, glands, and lymphoid tissue).

Valvulae conniventes create the characteristic small bowel fold pattern.

The superior mesenteric artery (SMA) supplies both the jejunum and ileum. A common small bowel mesentery anchors the jejunum and ileum to the posterior abdominal wall. The jejunum features larger, more feature-full folds and larger villi compared to the ileum.

153
Q

Small Bowel obstruction

A

Small bowel obstruction (SBO) is common and most often due to adhesions from prior surgery or hernia. Neoplasm, stricture, and intussusception are less common causes.

154
Q

Radiographic evaluation of small bowel obstruction

A

An abdominal radiograph is often the initial imaging evaluation for suspected obstruction.

Radiographic findings of SBO include small bowel distention and multiple air-fluid levels at different heights seen on the upright view. In addition, the lack of gas in the colon is especially suggestive of obstruction. (An upright or decubitus view is generally necessary to confidently diagnose obstruction.)

Potential false positives for diagnosing SBO on plain radiographs include: (Ileus with prior colectomy: Would not see gas in the colon. Ileus with ascites: Ascites often compresses the ascending and descending colon and rectum as these structures are not on a mesentery. However, gas in the transverse colon and sigmoid colon is still apparent.)

155
Q

CT Imaging of small bowel obstruction

A

CT is highly sensitive and specific for diagnosis of SBO. Small bowel distention >/= 3 cm with a transition point to collapsed bowel is highly specific for a small bowel obstruction.

In addition to diagnosing obstruction, CT can show the transition point, the cause of obstruction, and potential complications of obstruction such as ischemia or strangulation.

It is important to approach the interpretation of an obstruction in a systematic way.

First, look for the transition point to decompressed bowel to determine the cause.

Second, always determine if the obstruction is simple or closed-loop. A closed-loop obstruction is a never miss lesion as there is very high risk for bowel ischemia and severe morbidity and mortality.

Third, evaluate for signs of ischemia or impending ischemia, which include (in rough order of severity): Engorged mesenteric veins, Ascites surrounding the bowel (due to increased capillary permeability), Wall thickening (due to submucosal edema), Ladck of bowel wall enhancement (due to vasoconstriction or under-perfusion. Note that the presence or absence of bowel wall enhancement can only be assessed if positive oral contrast was not given.), Pneumatosis intestinalis (which is gas in the bowel wall due to necrosis. Pneumatosis produces multiple small locules of gas seen circumferentially in the bowel wall.

In addition to small bowel distention >3 cm and a transition point to decompressed bowel, an additional helpful CT finding of SBO is the small bowel feces sign, which describes feculent material mixed with gas bubbles in the small bowel that resembles the CT appearance of stool. (The small bowel feces sign is often seen just proximal to the transition point and is helpful to localize the site of transition. The small bowel feces sign may be especially helpful in subacute or partial obstruction, which can otherwise be difficult to diagnose. The small bowel feces sign is thought to be due to bacterial overgrowth and undigested food.

156
Q

Closed loop obstruction

A

Closed loop obstruction is a surgical emergency that may lead to bowel ischemia. Closed loop obstruction represents obstruction of both the efferent and afferent segments of a single loop of bowel.

Closed loop obstruction may be secondary to adhesions or hernia. The formation of a narrow pedicle can lead to volvulus, which predisposes to ischemia.

CT imaging features include a U-shaped distribution of the bowel loop with radially oriented vessels.If volvulus is present, the whirl sign may be seen, due to twisting of mesenteric vessels.

157
Q

Obstruction due to adhesions

A

Adhesions from prior surgery or intra-peritoneal inflammatory process are the most common cause of small bowel obstruction.

Adhesions are an imaging diagnosis of exclusion. On CT, a transition point is seen, but no obvious cause for the transition (e.g., no mass or hernia, etc) is identified.

The vast majority of patients with SBO due to adhesions have had prior abdominal surgery.

158
Q

Obstruction due to external hernia

A

Protrusion of bowel through the abdominal wall is the second most common cause of small bowel obstruction. Approximately 75% of external hernias occur in the groin, with the majority being inguinal hernias.

An inguinal hernia may be either indirect or direct, depending on the relation of the hernia to the inferior epigastic vessels.

  • Indirect: Indirect inguinal hernia it the most common type and is more common in males. The neck of the hernia is lateral to the inferior epigastric vessels. Hernia contents travel with the spermatic cord, often into the sctoum. Indirect inguinal hernias are considered a congenital lesion due to a patent processus vaginalis.
  • Direct: The neck of a direct inguinal hernia is medial to the inferior epigastric vessels, protruding through a weak are ain the anterior abdominal wall. The hernia contents do not go into the scrotum.

In an obturator hernia, bowel herniates through the obturator canal. Obturator hernias are almost always seen in elderly women due to pelvic floor laxity. (The key imaging finding is bowel located between the pectineus and obturator muscles. It is important to correctly diagnose an obturator hernia preoperatively. An obturator hernia requires a very different surgery from an inguinal hernia, and has an especially high morbidity and mortality if incarcerated.

Ventral hernia is often due to prior laparotomy.

159
Q

Obstruction due to internal hernia

A

Protrusion of bowel through the peritoneum or mesentery into a compartment in the abdominal cavity if a relatively uncommon cause of small bowel obstruction.

Transmesenteric hernia is a broad category of bowel herniation through defects in any of the three true mesenteries (small bowel mesentery, transverse mesocolon, and sigmoid mesentery). The most common type of transmesenteric hernia is the transmesocolic hernia, due to a defect in the transverse mesocolon (mesentery of the transverse colon). Transmesocolic hernia is seen most commonly post Roux-en-Y gastric bypass or biliary-enteric anastomosis from liver transplant.

  • The lack of confining hernia sac and variable imaging appearance make diagnosis difficult. A clue on imaging may be posterior displacement of the colon, with small bowel located anterior to the colon. The SMA and SMV may be displaced and engorged.
  • Internal hernias carry a high rate of volvulus. If volvulus is present, the whirl sign may be visible.
  • Transmesenteric hernias are also the most common type of hernia in children, not due to surgery but secondary to a congenital mesenteric defect thought to be grom prenatal intestinal ischemia. In children, the mesenteric defect has a variable position.

Paraduodenal hernia was previously the most common internal hernia (older literature states 53% of internal hernias were paraduodenal), prior to the rise in gastric bypass surgery. Paraduodenal hernias are congenital anomalies, due to embryologic failure of mesenteric fusion and resultant mesenteric defect. They more commonly occur on the left.

  • Paraduodenal hernia is associated with abnormal rotation of the intestine.
  • A common clinical complaint described by patients with paraduodenal hernia is chronic postprandial pain often relieved by massaging, which reduces the hernia.
  • In the more common left paraduodenal hernia, the bowel can herniate through a mesenteric defect named Landzert’s fossa, located behind the ascending (fourth) duodenum. The key imaging finding is a cluster of small bowel loops betwen the pancreas and stomach.

Foramen of Winslow hernia: The foramen of Winslow is the communication between the lesser sac and the greater peritoneal cavity. (The key imaging features of a foramen of Winslow hernia are dilated loops of bowel in the upper abdomen and presence of mesentery between the IVC and main portal vein.)

Obstruction due to neoplasm:

  • A mass intrinsic to the bowel or external compression from an extrinsic mass may cause small bowel obstruction. An extrinsic mass is usually straightforward to diagnose by CT.
  • Although the presence of an intraluminal mass may be more difficult to detect on CT, clues to the presence of an intrinsic mass include irregular bowel thickening and /or regional lymphadenopathy.
  • Primary small bowel neoplasm causing intrinsic bowel obstruction may be due to adenocarcinoma, GIST, and carcinoid. Metastatic causes of intrinsic bowel neoplasm include melanoma, ovarian, and lung cancer. Melanoma is known to cause intussusception.
  • Lymphoma is generally a “soft” tumor and rarely causes obstruction. Aneurysmal expansion of the small bowel wall is classic appearance, but presentation is highly variable.

Obstruction due to intussusception: While transient intussusceptions are a common incidental finding, an intussusception causing obstruction should raise suspicion for an underlying lesion and prompt surgery.

Obstruction due to Crohn disease: Stricture or active enteritis is an important cause of bowel obstruction in Crohn disease, especially the fibrostenotic subtype. Crohn disease is discussed on the following page.

Obstruction due to gallstone: Gallstone ileus is due to a gallstone that has eroded through into the small bowel, causing the classic Rigler’s triad of pneumobilia (from cholecystoduodenal fistual), small bowel obstruction, and ectopic gallstone within the small bowel.

160
Q

Enteritis

A

Enteritis is inflammation of the small bowel. The most common CT manifestation of enteritis is bowel wall thickening. Mesenteric stranding or free fluid may also be present.

161
Q

Crohn Disease

A

Crohn disease is a chronic granulomatous inflammatory condition that may affect any part of the gastrointestinal tract from the mouth to the anus. Involvement is discontinous, with characteristic skip lesions of intervening normal GI tract. The most common site of involvement is the small bowel, especially the terminal ileum.

The earliest histologic changes occur in the submucosa, seen on imaging as aphthous ulcers due to lymphoid hyperplasia and lymphedema.

Endoscopy and barium fluoroscopy (small bowel follow-through, enteroclysis, and barium enema) have historically been the modalities to evaluate Crohn disease. More recently, however, CT and MR enterography are emerging as the exams of choice.

  • The advantages of CT nad MRI are the ability to visualize beyond the bowellumen to evaluate the bowel wall, presence of extraintestinal complications, and the vasculature.
  • The disadvantages of CT and MRI compared to fluoroscopy and endoscopy are reduced spatial resolution and limited sensitivity for detecting subtle early signs of disease.

The most common imaging finding on all modalities is wall thickening of the terminal ileum.

Fluoroscopic findings include thickened, nodular folds in the affected regions of small bowel, luminal narrowing, mucosal ulceration, and separation of bowel loops. The typical cobblestone appearance seen on endoscopy and fluoroscopy is a result of crisscrossing deep ulcerations.

The fibrostenotic subtype of Crohn disease may clinically present with bowel obstruction. Asymmetric bowel fibrosis from ulcerations of the mesenteric side of the bowel produces pseudosacculations on the antimesenteric side. The fibrosis can lead to a segmental stricture, called the string sign.

Complications of Crohn disease include bowel stricture, fistulae, and abcesses.

162
Q

Scleroderma

A

Scleroderma is a systemic disease characterized by the deposition of collagen into multiple internal organs and the skin.

The primary insult to the gastrointestinal tract in scleroderma is imparied motility due to replacement of the muscular layers with collagen, which leads to slowed transit and subsequent bacterial overgrowth, progressive dilation, and pseudo-obstruction.

Radiographic findings are sacculations on the antimesenteric border (side opposite where the mesentery attaches) and a hidebound bowel due to thin, straight bowel folds stacked together.

Treatment is with antibiotics for bacterial overgrowth and prokinetic drugs such as erythromycin or octreotide for bowel motility.

163
Q

Celiac disease (sprue, gluten-sensitive enteropathy)

A

Celiac disease, also known as sprue and gluten-sensitive enteropathy, is an autoimmune, proximal enteritis caused by a T-cell-mediated immune response triggered by antigens in ingested gluten.

The primary sites of involvement are the duodenum and jejunum.

The most characteristic imaging finding of celiac disease is reversal of jejunal and ileal fold patterns. Normally, the jejunum has more folds than the ileum. However, in celiac disease, the loss of jejunal folds causes a compensatory increase in the number of ileal folds.

Villous atrophy causes the loss of jejunal folds and hypersecretion of intraluminal fluid that creates flocculations of barium due to lack of contrast adhesion to the bowel wall. The moulage (French for casting) sign is seen on a barium study and refers to a cast-like appearance of the featureless jejunum.

The CT findings of celiac disease include dilated, fluid-filled bowels, often with intra-luminal flocculations of enteric contrast. Contrast can be seen both insinuated between the small bowel folds and centrally within the bowel, with a peripheral layer of low-attenuation secretions. Other CT findings of celiac disease include mesenteric adenopathy and engorgement of mesenteric vessels.

Unlike other causes of enteritis, diffuse bowel wall thickening and ascites are less common.

An important complication of celiac disease is small bowel T-cell lymphoma, which may manifest as an exophytic mass, circumferential bowel wall thickening, or enlarged mesenteric lymph nodes.

Other complications of celiac disease include:

  • Intussusception, thought to be due to uncoordinated peristalsis, without a lead-point mass
  • Pneumatosis intestinalis, thought to be due to dissection of intraluminal gas through the inflamed bowel wall. Pneumatosis in the setting of celiac disease is not thought to reflect bowel ischemia.
  • Splenic atrophy.
  • Increased risk of venous thromboembolism
  • Lab abnormalities include anemia (secondary to malabsorption), leukopenia, and immunoglobulin deficiency. Skin abnormalities include the characteristic dermatitis herpetiformis rash.
  • Cavitating mesenteric lymph node syndrome (CMLNS) is a very rare complication of celiac disease, with only 36 reported cases in the literature. The central portion of the affected lymph nodes shows low attenuation due to liquid necrosis. CMLNS is thought to be highly specific for celiac disease when seen in combination with villous atrophy and splenic atrophy. The differential diagnosis of low attenuation mesenteric lymph nodes includes tuberculosis, Whipple disease, treated lymphoma, and CMLNS.
164
Q

Infectious enteritis

A

Several bacterial, viral, and fungal organisms may cause enteritis.

Yersinia and tuberculosis have a propensity to affect the terminal ileum, mimicking Crohn disease.

Salmonella is the most common cause of food-borne gastroenteritis and causes segmental distal small bowel thickening on CT and segmental nodular thickened folds on fluoroscopy.

165
Q

Radiation enteritis

A

Long-term effects of radiation to the pelvis include adhesive and fibrotic changes to the mesentery and small bowel.

Clues to the diagnosis of radiation enteritis include a history of radiation therapy and regional involvement of bowel loops not confined to a vascular territory.

Imaging findings include mural thickening and mucosal hyperenhancement with narrowing of the lumen. Radiation enteritis may be a cause of small bowel obstruction.

166
Q

Whipple disease

A

Whipple disease is due to infection by Tropheryma whippelii, which manifests in the GI tract as malabsorption and abdominal pain. Whipple disease may cause arthralgias and increased skin pigmentation.

Whipple disease characteristically causes low attenuation adenopathy that may appear similar to the cavitating mesenteric lymph node syndrome seen in celiac disease.

Radiographically, Whipple disease causes thickening and nodularity of duodenal and proximal small bowel folds. In contrast to celiac disease, there is typically no hypersecretion.

167
Q

Graft versus host disease (GVHD)

A

Graft versus host disease is a complication of bone marrow transplantation. The skin, liver, and gastrointestinal tract are most commonly affected.

Imaging findings of GVHD include nonspecific wall thickening and effacement of the normal small bowel fold pattern. While the classic bariumfinding is the ribbon bowel, this is not often seen.

168
Q

Coltits overview

A

Colitis is inflammation of the colon that may be caused by several unrelated etiologies, often with overlapping imaging findings.

The primary imaging features of colitis is bowel wall thickening. Generally a full clinical evaluation, stool studies, and sometimes colonic biopsy are required for a definitive diagnosis.

Incidental colonic wall thickening is found in as many as 10% of CT scans.

169
Q

Ischemic colitis

A

Colonic ischemia can be caused by acute arterial thrombus, chronic arterial stenosis, low-flow states (e.g., congestive heart failure), and venous thrombosis.

The splenic flexure is the watershed region between the superior and inferior mesenteric arteries and is especially susceptible to ischemia in low-flow states.

The rectum is supplied by a dual blood supply and is almost never affected by ischemia. The superior rectal artery (terminal branch of the IMA) and the inferior and middle rectal arteries (arising from teh internal iliac artery anterior division) form perirectal collaterals.

A suggestive CT finding of ischemic colitis is segmental, continuous thickening of the affected colon in a vascular distribution, with sparring of the rectum.

If arterial thromboembolic disease is suspected, one should evaluate for the presence of aortic atherosclerotic disease or a left atrial thrombus in the setting of atrial fibrillation.

If chronic arterial stenosis is suspected, one should evaluate for atherosclerosis of the mesenteric vessels.

170
Q

Infectious colitis

A

Infectious colitis can be bacterial, tubercular, viral, or amoebic. There is a large overlap in the clinical presentation and imaging findings of the various pathogens.

In general, infectious colitis features pericolonic stranding and ascites in addition to the colonic wall thickening seen in all forms of colitis.

Yersinia, Salmonella, and colonic tuberculosis affect the right colon. Tuberculosis is known to involve the ileocecal valve, resulting in a desmoplastic reaction that mimics Crohn disease.

E. coli, CMV, and C. difficile colitis (discussed below) most commonly cause pancolitis

171
Q

Pseudomembranous colitis

A

Pseudomembranous colitis is an especially prevalent form of infectious colitis caused by overgrowth of Clostridium difficile, most commonly due to alteration in colonic bacterial flora after antibiotc use. Pseudomembranous coltiis may also occur without a history of antibiotics, especially in hospitalized or nursing home patients.

A key imaging finding is marked thickening of the colonic wall, typically with involvement of the entire colon (pancolitis). The accordion sign describes severe colonic wall thickening combined with undulation of enhancing inner mucosa. It signifies severe colonic edema but is not specific to C. difficile. Thumbprinting is a fluoroscopic finding of thickened haustra and is also due to edema.

172
Q

Ulcerative colitis

A

Ulcerative colitis (UC) is an idiopathic inflammatory bowel disease that begins distally in the rectum and spreads proximally in a continual manner (unlike Crohn disease, which features skip areas). Of note, it is possible for the rectum to appear normal with more proximal colonic involvement present if the patient has been treated with corticosteroid enemas.

Patients with UC have an increased risk of primary sclerosing cholangitis, colon cancer, and cholangiocarcinoma.

Extra-abdominal manifestations of UC include sacroilitis, iritis, erythema nodosum (tender red subcutaneous nodules), and pyoderma gangrenosum (cutaneous ulcers).

UC does not extend more proximally than the cecum; however, a backwash ileitis caused by reflux of inflammatory debris into the ileum may mimic Crohn disease.

Imaging of ulcerative colitis features circumferential wall thickening with a granular mucosal pattern that is best seen on barium enema. Pseudopolyps may be present during acute inflammation, representing islands of normal mucosa surrounded by inflamed mucosa. A collar-button ulcer is nonspecific but represents mucosal ulceration undermined by submucosal extension.

Chronic changes of ulcerative colitis include a featureless and foreshortened lead pipe colon. Similar to Crohn disease, fat-attenuation of the colonic wall suggests chronic disease, as seen in the case above.

Toxic megacolon is a severe complication of ulcerative colitis (and less commonly, Crohn disease) caused by inflammation extending through the muscular layer. Imaging of toxic megacolon shows dilation of the colon to greater than 6 cm in assocation with an adynamic ileus. Colonic perforation may occur and colonoscopy is contraindicated in suspected toxic megacolon.

173
Q

Typhlitis (neutropenic enterocolitis)

A

Typhlitis is a right-sided coltisi seen in immunocompromised patients.

Treatment is with broad-spectrum antibiotics and antifungals.

174
Q

Familial Adenomatous Polyposis (FAP)

A

Familial adenomatous polyposis (FAP) is an autosomal-dominant syndrome featuring inumerable premalignant adenomatous polyps in the colon and to a lesser extent the small bowel. Prophylactic colectomy is the standard of care to prevent colon cancer.

Gastric polyps are also present, although the gastric polyps are hyperplastic and are not premalignant.

Gardner syndrome is a variant of FAP. In addition to colon polyps, patients also have: (Desmoid tumors, Osteomas, Papillary thyroid cancer, Epidermoid cysts).

Turcot syndrome is another variant of FAP. In addition to colon polyps, patients also have CNS tumors (gliomas and medulloblastomas).

175
Q
A
176
Q

Hereditary nonpolyposis colon cancer syndrome (HNPCC) = Lynch syndrome

A

Hereditary nonpolyposis colon cancer (HNPCC) syndrome (also called Lynch syndrome), is an autosomal dominant polyposis syndrome caused by DNA mismatch repair, leading to colon cancer from microsatellite instability on a molecular level.

Similar to FAP, the colon polyps of HNPCC are adenomatous.

HNPCC is associated with other cancers, including endometrial, stomach, small bowel, liver, and biliary malignancies.

177
Q

Peutz-Jeghers

A

Peutz-Jeghers is an autosomal dominant syndrome that features multiple hamartomatous pedunculated polyps, usually in the small bowel. These polyps may act as lead points and cause intussusception.

Characteristic skin manifestations include perioral mucocutaneous blue/brown pigmented spots on the lips and gums.

Peutz-Jeghers is associated with gynecologic neoplasms as well as gastric, duodenal, and colonic malignancies.

178
Q

Cowden syndrome

A

Cowden syndrome is an autosomal dominant syndrome of multiple hamartomatous polyps most commonly found in the skin and external mucous membranes, but also in the gastrointestinal tract.

Cowden syndrome is associated with an increased risk of thryoid cancer (usually follicular),as well as skin, oral, breast, and uterine malignancies.

179
Q

Cronkhite - Canada

A

Cronkhite-Canada is a non-inherited disorder (the ony polyposis syndrome in this list that is not autosoma dominant) consisting of hamartomatous polyps throughout the gastrointestinal tract.

Cutaneous manifestations include abnormal skin pigmentation, alopecia, and onychodystrophy (malformation of the nails).

180
Q

Appendicitis

A

Appendicitis is the most common surgical cause of acute abdomen. Acute inflammation of the appendix is thought to be due to obstruction of the appendiceal lumen, leading to venous congestion, mural ischemia, and bacterial translocation.

Appendicitis represents a spectrum of severity ranging from tip appendicitis (inflammation isolated to the distal appendix) to gangrenous appendicitis with abscess if the disease is not diagnosed until late.

Greater than 97% of patients undergo a preoperative CT prior to appendectomy, with resultant decrease in negative appendectomy rate from 23% in 1990 to 1.7% in 2007.

Imaging of appendicitis relies on direct and indirect imaging findings.

Direct findings of appendicitis are due to abnormalities of the appendix itself: (Distended, fluid-filled appendix: 6 mm is used as cutoff for normal diameter of the appendix, although there is wide normal variability and 6 mm is from the ultrasound literature using compression. A normal appendix distended with air can measure >6 mm; therefore, some authors advocate using caution with a numeric cutoff in an otherwise normal-appearing appendix filled with air or enteric contrast. Appendiceal wall-thickening. Appendicolith, which may be cuase of luminal obstruction; however, appendicoliths are commonly seen without associated appendicitis.)

Indirect findings of appendicitis are due to the spread of inflammation to adjacent sites: (Periappendiceal fat stranding, Cecal wall thickening, Hydroureter, Small bowel ileus.)

Appendicitis can also be evaluated by ultrasound, with the key sonographic finding a tubular, blind-ending, non-compressible right lower quadrant structure measuring > 6 mm in diameter. It is generally necessary to use graded compression to evaluate for compressibilty.

181
Q

Diverticulitis

A

Diverticulitis is microperforation and acute inflammation of a colonic diverticulum. CT is the primary modality for diagnosis, triage, and evaluation of severity and complications.

The left colon is affected far more commonly than the right.

It is often impossible to distinguish acute diverticulitis from microperforated colon cancer. Many authors recommend follow-up colonoscopy after the acute episode has resolved, although this recommendation is somewhat controversial and varies by institution.

Uncomplicated diverticulitis does not have any imaging evidence of bowel perforation (even though histopathologically all diverticulitis is associated with bacterial translocation across the bowel wall). CT findings of uncomplicated diverticulitis include bowel wall thickening and pericolonic fat stranding, usually centered around a culprit diverticulum.

Complicated diverticulitis implies the presence of an additional complication, including: Pericolonic or hepatic abscess. Extraluminal air. Bowel obstruction. Bowel fistula (colovesical fistula most common, apparent on imaging as gas in the bladder not explained by Foley catheter placement). Mesenteric venous thrombosis.)

Uncomplicated diverticulutis is typically treated conservatively.

Abscesses can usually be drained percutaneously.

Indications for surgery include the presence of a fistual or recurrent diverticulitis, with two prior episodes of diverticulitis treated conservatively.

182
Q

Epiploic appendagitis

A

Epiploic appendagitis is a benign, clinical mimic of diverticulitis caused by torsion of a normal fatty tag (appendage) hanging from the colon.

Epiploic appendagitis has a pathognomonic imaging appearance of an oval fat-attenuation lesion abutting a normal colonic wall, with mild associated fat stranding. A central hyperdense dot in cross-section represents the thrombosed central vein of the epiploic appendage.

Treatment is with anti-inflammatories, not antibiotics or surgery.

183
Q

Mesentery and Peritoneum Anatomy

A
184
Q

Peritoneum

A

The peritoneum is a thin membrane consisting of a single layer of mesothelial cells that are supported by subserosal fat cells, lymphatic cells, and white blood cells.

The visceral peritoneum lines the surface of all intraperitoneal organs, while the parietal peritoneum lines the outerwalls of the peritoneal cavity.

The most dependent portion of the peritioneal cavity (both supine and upright) is the pouch of Douglas in women and the retrovesical space in men.

185
Q

Mesentery

A

There are three true mesenteries, while each supply a portion of the bowel and connect to the posterior abdominal wall. Each mesentery consists of a network of blood vessels and lymphatics, sandwiched between the peritoneal layers. The three true mesenteries are: Small bowel mesentery: Supplies both the jejunum and ileum. Oriented obliquely from the ligament of Treitz in the left upper quandrant to the ileocecal junction in the right lower quadrant. Transverse mesocolon: Mesentery to the transverse colon, connecting the posterior transverse colon to the posterior abdominal wall. Sigmoid mesentery: Mesentery to the sigmoid colon.)

The greater and lesser omentum are specialized mesenteries that attach to the stomach. The greater and lesser omentum do not connect to the posterior abominal wall.

Greater omentum: Large, drape-like mesenter in the anterior abdomen, which connects the stomach to the anterior aspect of the transverse colon.

Lesser omentum: Connects stomach to liver.

186
Q

Flow of peritoneal fluid

A

Peritoneal fluid is constantly produced, circulated, and finally resorbed around the diaphragm, where it eventually drains into the thoracic duct.

187
Q

Overview of the “misty” mesentery

A

As previously discussed, the abdominal mesenteries are fatty folds through which the arterial supply and venous and lymphatic drainage of the bowel run.

The mesenteries themsevles are not seen on CT because they are made primarily of fat and blend in with intra-abdominal fat. However, the vessels which course through the mesentery are normally seen.

Infiltration of the mesentery by fluid, inflammatory cells, tumor, or fibrosis may increase the attenuation of the mesentery and cause the mesenteric vasculature to appear indistinct. These findings are often the first clue to certain pathologies.

188
Q

Mesenteric edema

A

Edema of the mesentery may be secondary to either systemic or intra-abdominal etiologies.

Systemic causes of edema include congestive heart failure, low protein states, and third-spacing, all of which can lead to diffuse mesenteric edema.

Focal mesenteric edema may be secondary to an intra-abdominal vascular cause, such as mesenteric vessel thrombosis, Budd-Chiari syndrome, or IVC obstruction. Abdominal vascular insults may cause bowel ischemia, which manifest on imaging as bowel wall thickening, pneumatosis, or mesenteric venous gas.

189
Q

Mesenteric inflammation

A

The most common cause of mesenteric inflammation in the upper abdomen is acute pancreatitis. However, any focal inflammatory process such as appendicitis, inflammatory bowel disease, and diverticulitis may cause local mesenteric inflammation leading to the “misty” mesentery appearance.

Mesenteric panniculitis is an idiopathic condition, which may cause a diffuse “misty” mesentery.

190
Q

Intra-abdominal hemmorrhage

A

Intra-abdominal hemorrhage tends to be localized, surrounding the culprit bleeding vessel unless large. Hemorrhage may be secondary to trauma, post-procedural, or due to anticoagulation.

191
Q

Neoplastic infiltration

A

Neoplastic infiltration of the mesentery may cause the “misty” mesentery. The most common tumor involving the mesentery is non-Hodgkin lymphoma, which typically also causes bulky adenopathy.

Mesenteric involvement may be especially apparent after treatment, where the “misty” mesentery is limited to the portion of the mesentery that contained the treated lymph nodes.

Other tumors that may involve the mesentery include pancreatic, colon, breast, gastrointestinal stromal tumor, and mesothelioma.

192
Q

Carcinoid Mesenteric Mass

A

Gastrointestinal carcinoid is relatively rare tumor compared to other gastrointestinal malignancies, but is the most common small bowel tumor. It typically occurs in the distal ileum.

Carcinoid usually arises as an intraluminal mass and may secondarily spread to the mesentery either by direct extension or lymphatic spread. Up to 80% of carcinoids spread to the mesentery.

A classic imaging appearance of carcinoid affecting the mesentery is an enhancing soft-tissue mass with radiating linear bands extending into the mesenteric fat. Calcification is common. The radiating linear bands do not represent infiltrative tumor but are the result of an intense desmoplastic reaction caused by the release of serotonin by the tumor.

The differential diagnosis of a sclerosing mesenteric mass includes: Carcinoid, Desmoid tumor, Sclerosing mesenteritis.

193
Q

Desmoid tumor

A

Desmoid tumor is a benign, locally aggressive mass composed of proliferating fibrous tissue.

Desmoid may be sporadic, but mesenteric desmoid tumors are more common in patients with Gardner syndrome (a variant of familial adenomatous polyposis).

On CT, most desmoids are isoattenuating to muscle, but large tumors may show central necrosis. A characteristic imaging feature is strands of tissue radiating into the adjacent mesenteric fat, similar to mesenteric carcinoid and sclerosing mesenteritis.

194
Q

Sclerosing mesenteritis

A

Sclerosing mesenteritis is a rare inflammatory condition that leads to fatty necrosis and fibrosis of the mesenteric root.

Imaging of sclerosing mesenteritis shows mesenteric masses with striations of soft tissue extending into the adjacent fat. Calcification may be present.

Mesenteric panniculitis is a variant where inflammation predominates and presents as acute abdominal pain. On CT, there is a “misty” mesentery, sometimes with linear bands of soft tissues representing early fibrosis.

195
Q

Mesenteric metastases and lymphoma

A

Gastric, ovarian, breast, lung, pancreatic, biliary, colon cancer, and melanoma can metastasize to mesenteric lymph nodes.

Mesenteric lymphoma can produce the sandwich sign, where the mesenteric fat and vessels (the sandwich filling) are engulfed on two sides by bulky lymphomatous masses (the bread).

196
Q

Peritoneal Carcinomatosis

A

Peritoneal carcinomatosis represents disseminated metastases to the peritoneal surface.

The term omental caking describes the replacement of omental fat by tumor and fibrosis.

Mucinous adenocarcinoma is the most common tumor type to cause peritoneal carcinomatosis. Peritoneal carcinomatosis due to mucinous adenocarcinoma should not be confused with psuedomyxoma peritonei, discussed on the following page.

197
Q

Pseudomyxoma Peritonei

A

Pseudomyxoma peritonei is a low-grade malignancy characterized by copious mucus in the peritoneal cavity.

In general, pseudomyxoma peritonei is thought to be produced by a mucin-producing adenoma or adenocarcinoma of the appendix; however, there is some controversy as to whether the ovary or colon can be a primary site as well. Pseudomyxoma peritonei is often associated with an ovarian mass (up to 30% of female patients), but it is thought that these are most often metastatic deposits.

Pseudomyxoma peritonei was previously thought to be produced by a benign appendiceal mucocele, which is now believed to occur much less commonly than originally thought. 20% of all appendiceal adenomas or adenocarcinomas will cause pseudomyxoma peritonei. Only 2% of all appendiceal mucoceles (which occur slightly less commonly than appendiceal adenomatous lesions) will cause pseudomyxoma peritonei.

Tumor deposits tend to be spread throughout the entire peritoneal cavity due to intraperitoneal fluid currents.

Clinically, pseudomyxoma peritonei presents with recurrent mucinous ascities. The surgeons refer to the mucinous ascites as a “jelly belly”.

CT shows lobular ascites that is typically of slightly higher attenuation (5-20 hounsfield units) compared to fluid ascites. Occasionally, mucus can be seen in the region of the appendix, but the flow of peritoneal contents tends to spread the mucinous ascites diffusely throughout the peritoneum.

Advanced disease shows pathognomonic scalloping of the hepatic margin.

Treatment continues to evolve, but the best outcomes are primarily with surgical treatment and hyperthermic intraperitoneal chemotherapy lavage.